You are on page 1of 54

READING COMPREHENSION TESTS

EX 1. Read the following passage and the decide which option best fits each space.
Have you ever stopped (1)……… why people give each (2)……….. eggs at Easter? - The Christian
festival of Easter celebrates the return of Christ from the dead, but the festival is actually named (3)……….. the
goddess of the sun, Eostre, whose name is taken from the East where she (4)……….. In very ancient times, Easter
was a celebration that winter was (5)……… and that a new life was about to begin. The rabbit, (6)……….. to the
number of young it produces, it the symbol of life. In some parts of the world, the rabbit leaves large (7) …….. of
eggs (another symbol of new life ) in the garden and children have to find as many as they can. This is very (8)
…………. Christmas when Santa Claus leave presents for individual children. At Easter children have to be
independent and (9)……… after themselves. In this (10)……….. the hunt for Easter eggs presents the need for
young people to go out in to the world and make their own fortune.
1A. to wonder B. wondering C. wonder D. wander
2A.else B. person C. others D.other
3A.for B. about C. after D. with
4A. goes B. sets C. rises D. raises
5A. finish B.conclusion C. up D. over
6A. as B.since C. due D. because
7A. numbers B. sums C. fingures D. totals
8A. like B.unlike C.different D.similar
9A.take B.get C.look D. carry
10A day B. way C.habit D.time
EX 2. Read the passage carefully and choose the best answer to the questions below ;
During the teenage years, many young people can at times be difficult to talk to. They often seem to
dislike being questioned. They may seem unwilling to talk about their work in school. This is a normal
development at this age, though it can be very hard for parents to understand. It is part of becoming independent of
teenagers trying to be adult while they are still growing up. Young people are usually more willing to talk if they
believe that questions are asked out of real interest and not because people are trying to check up on them.
Parents should do their best to talk to their sons and daughters about school work and future plant but
should not push them to talk if they don't want to. Parents should also watch for the danger signs: some young
people in trying to be adult may experiment with sex, drugs, alcohol, or smoking. Parents need to watch for any
signs of unusual behavior which may be connected with these and get help if necessary.
1.This passage is taken from a
A. handbook for parents B. school timetable
C. teenage magazine. C. book for children
2. Why do adults sometimes find teenagers difficult to talk to.
A. because most teenagers are quiet
B. because teenagers don't want to talk to other people.
C. because teenagers think adults are not honest
D. because most teenagers hate adults.
3. When can you expect young people to be more talkative than usual.
A. When people talk to them because they are really interested and not just checking on them.
B. When adults give them a lot money to spend.
C. When adults talk to them about something other than their work in school.
D. When adults talk to them about sex, alcohol and drugs.
4. Some teenagers experiment with drinking bad smoking because
A. cigarettes and alcohol are available everywhere.
B. cigarettes and alcohol are cheap
C. women like smoking and drinking men
D. they regard them as a mark of adulthood
5. The word BEHAVIOR in the passage most nearly means
A. feeling B. manners
C.activities D. reaction.

1
EX 3. Fill in the blanks in the following passage with ONE suitable word.
Watching Television and going for a walk are the most popular leisure (1)................. in Britain. But although
longer holidays and shorter working hours have given people more free (2)............, women generally have less
free time (3)............ men, because they spend time (4)................. domestic work, shopping and childcare..
Surveys showed that more men (5).................. newspapers than women, and (6).............. slightly higher
proportion of adult read Sunday newspapers than read (7).................... morning national newspaper.
More people are taking holidays abroad (8)............... 1971 only 36 percent of (9).............. in Britain had been
abroad on holiday; but by 1983 this proportion had (10)................. to 62 percent, nearly fifteen million people.

EX 4. Fill in each blank with an appropriate word or phrase from the box.
HOW TELEVISION AFFECTS OUR LIVES
more which television that than programs pictures news viewers people

Television is an important invention of the twentieth century. It has been so popular (1) …………. now we
can’t imagine what life would be like if there were no (2) …………….... Television is a major means of
communication. It brings (3) ………………… and sounds from around the world into millions of home. Through
television, (4) ……………… can see and learn about people, places and things an faraway lands. Television
widens our knowledge by introducing us to new ideas (5) ……………….. may lead us to new hobbies and
recreation. In addition to the (6) …………………, television provides us with variety of (7) ………………. that
can satisfy every taste. Most (8) …………….. now like to spend their evenings watching television. It is (9)
…………………. convenient for them to sit at home watching television (10) ………………… to go out for
amusements elsewhere.
EX 5. Choose the best answer:
There has been a revolution in the world of newspapers. Not many years ..1....., newspapers were being
produced using techniques unchanged for ..2.. hundred years.
The journalists gave their stories to a typist, who prepared them for an editor, who passed them on...3... the
printer. The printer who was a ..4.... skilled man, set up the type. ......5... was then collected to make the pages.
When the pages were complete, the printing machines could be...6.......
Nowadays what ...7.....? The journalists type their stories into a computer. The ...8 ..... checks their
spelling, plans the page, shapes the articles. When the pages are ready, another computer may control the printing.
........9.. can be no doubt about it, producing a newspaper in an entirely different ..10.......now
1.a.before b. after c.ago d. yet
2.a.a b.some c.an d.over
3.a.to b.by c.through d.with
4.a.hardly b.mostly c. partly d.very
5.a.They b.Which c.This d.All
6.a.switched b.started c.stopped d.moved
7.a.gives b.occurs c.goes d.happens
8.a.computer b.editor c.typist d.printer
9.a.It b.There c.You d.We
10.a.skill b.work c.management d.business
EX 6. Fill in each numbered blank with a suitable word
When you hear the word "bank", what do you think of ? A .........(1) to put money? The land on the edge of a
river? To depend on something or someone ? If you choose any of .....
............(2) things, you're right. Why ? because words often have .............(3) than one meaning. The tricky part is
figuring out ...........(4) meaning is appropriate.
Choosing the correct meaning of a word depends on .............(5) things. First, it depends on the words
and ............(6) surrounding your particular word. The other words and sentences give you context
clues. .................(7), the meaning depends on how the word is ............(8) in the sentence. Is it a noun, a
verb, ................(9) adjective or part of a phrase? Knowing the word's part of speech will ..............(10) you
discover the word's precise meaning.
EX 7. Read the text then choose the correct answer
Computing is now at the same stage as printing was when the first printing presses were used. Before printing
presses were invented, only rich people like kings and dukes could afford to buy books. Often these people were
unable to read and hadn't enough time to learn. In any case, the books were so big that it was difficult for anyone

2
to relax with a book as we do today. They wanted books because they were expensive and there was something
magical about them. Only a few people were able to write, and it took an extremely long time to write a book.
Monks and other people who could write said ordinary people could not learn to read.
The position with computers in very similar today. A few years ago, computers were very large and expensive.
Business managers and rich people ordered them but they didn’t know how to use them. In many countries,
however, the situation has now completely changed. Lots of people not only own microcomputers but also know
how to use them.
1. What happened before printing presses were invented ?
a. Books were so big that people didn’t want to buy them.
b. Only intelligent and rich people could read books.
c. Only kings and dukes had enough money to buy books.
d. All rich people bought books.
2. Why were books wanted before printing presses were invented ?
a. to show that the people who owned the books were rich and intelligent.
b. to show that the people who owned the books could write.
c. Because books were dear and magical.
d. Because books showed that the person who owned them were magical people.
3. What is the common feature of a book and a computer?
a. They were only for intelligent people. b. They were too expensive to buy.
c. They were large and expensive at first. d. Only people who could use them buy them.
4. What situation has thoroughly changed nowadays?
a. A lot of people are eager to buy computers b. A lot of computers are sold
c. Even children can use a computer d. A person who buys a computer knows how to use it.
EX 8. Fill each of the numbered blanks in the following passage. Use only one word in each space.
Population growth is a serious (1)...........around the world. At the beginning of the 20 th (2)...........,there
were about 1.5 billion (3)........ in the world. In 1984 the world population (4)..........4.8 billion people. By the year
2000, ...........(5) Will be about 6.1 billion. This growth in population is not happening (6).......... For example in
Europe the population is not growing at all (7).........in these countries are smaller now. Only about 2.1 (8).........are
born for every woman.
EX 9. Put one suitable word in each space.
I'm a writer, and I work ......1....... home. It doesn't particularly ........2....... where my house is, as long as it
is ......3....... nice house. But last year my wife ........4..... a job in a different part of the country. So, we had to
move. This was our first ..........5........ of ......6....... one house and selling another at the same time. I wouldn't like
to do it again. We quickly saw the house .........7........ we wanted. It was then that our problems began: The people
who wanted to buy our house couldn't sell ........8......... and the man ........9..... house we wanted couldn't find a
house he liked. It ........10......... us nine months to buy the house. Now, I'm not sure if I like it.
EX 10. Fill in each blank with one suitable word.
The Browns live in the countryside. Last week when they came up to London, they …(1)… their times in
seeing as much as …(2)… . Mrs. Brown was most interested in stores, especially in Oxford Street, and bought
some of …(3)… things which she could not find in the shops in the countryside. Their two children, George and
Susan, …(4)… had never been to London before, were surprised …(5)… the crowds everywhere. They enjoyed
traveling on the underground and going down the moving stairs …(6)… lead to the platforms.
One day they went to Hyde Park and walked along on the grass to Kensington Garden …(7)… the bright autumn
sunshine. It was very quiet here. Only …(8)… noise from the streets reminded them that they were still in a city.
The …(9)… of the tall trees were turning red, brown and yellow, birds were …(10)… about, sheep …(11)…
eating the grass, there were …(12)… of flowers, and ducks were swimming about on the Round Pond. George, …
(13)… hobby was planes and boats, was very much interested in the model boats which boys were sailing …(14)
… the Pond.
Before the end of the week they had seen a great …15… . They hoped, however, to see more of London on
their next visit.
EX 11. Fill in each blank with one suitable word.
John Lennon was born in Liverpool in 1940. He was always (1)............. on music and played in pop groups
(2)............school and Art College. John got married (3).................... Cynthia in 1962 and they had a son,
(4)...........name was Julian. At that (5)..........., John was member of a group (6) ..........."The Beatles". Many
beautiful songs (7)........... written by John and wherever the group went, crowds of fans gathered to see them.

3
They (8) ................ scream and faint when "The Beatles" played, and lot of people (9)...........their hair cut in a
Beatles styles. Soon everyone had heard of "The Beatles" and John was (10) ...........richer than he had ever been.
EX 12. Read the passage and then answer the questions below.
Alfred Bernard Nobel (1833 - 1896) a Swedish scientist was the first man to make dynamite. He allowed his
invention to be used in peace. However, he saw it being used in war to destroy things, he was extremely unhappy.
On his death, he left all his money to be spent upon a prize every year. The Nobel Prize is now one of the greatest
prizes that a person can ever receive. It is given every year for the best work in one of the following subjects:
Physics, Chemistry, Medicine; Literature and Peace. Some of the world's greatest scientists are asked to choose
who should be given the prizes.
1. What was Nobel's invention about?
2. How was he when he knew his invention being used for bad purpose?
3. Did he give all the money to his relatives on his death?
4. What is his money used for?
5. Can we choose who should be given the prizes? If not, who can?
EX 13. Fill in each blank with one suitable word.
Most ...1.. the addicts are men. They come home ...2...... work, eat their meal quickly and then spend the
evening ....3....... their computers. Some of them make programs, ...4...... most of them just play games. Some
wives say the computer is killing their marriage. Their husbands play until three.....5....four o'clock ....6...... the
morning and all weekend. People call these lonely wives "computer widows".
When television became popular ...7...... the 1950s, doctors said it caused "television neck", "TV eyes" and
other new illnesses. Now it is the home computer. People say it causes headaches, backaches and makes their eyes
tired. But worst ....8...... all, it is addictive. That means it is ..9.... drinking, smoking or taking drugs. Some people
can't stop ...10.....it.
EX 14. Read the passage and choose the best answer.
An important part of world history is the story of communication. In prehistoric times, for example, people
did not have books. They did not know much about geography. People were limited. They knew only about
themselves and their environment (the land around their homes). Their knowledge of geographical things like
mountains and rivers was limited. They did not travel very far. Sometimes they knew about nearby people and
communicated with them. They sent messages in simple ways. Early signals for communication included
smoke from fires and the sounds of drums.
Then people formed towns, and then cities, as safe places to live. Soon they began to develop other ways to
communicate, to spread information. People began to buy and sell things to one another. Because of their
business, they developed writing systems to keep records and to send messages. Life was changing for many
people. Business caused changes, and so did the invention of the printing press. Many more people learned to
read then.
Suddenly communication and knowledge improved greatly. People sent letters and news by horse and carriage.
Later the mail went by train and then by airplane. World communication was now a possibility.
1. Prehistory was ......................
a. 200 years ago, when we had history books.
b. many years ago, before history books were written.
c. after the development of the printing press.
2. People live in their .........................
a. horse carriage b. own environments c. Businesses.
3. Mountains and rivers are geographical things. So is .................
a. smoke b. a drum c. the ocean
4. Many more people learned to read ..............the invention of the printing press.
a. during b. before c. after
5. the main idea of the reading text
a. An important part of world history is the story of communication.
b. In prehistoric times, for example, people did not have books.
c. Then people formed towns, and then cities, as safe places to live.

4
EX 15. Read the paragraph. Then choose the best answer for each question.
Two popular traditions at Christmas are : decorating the home and singing the Christmas carols. The home is
the center of the Christmas celebration. Inside, an evergreen tree is usually placed in the corner of the living room.
Children and their parents wrap string of colorful lights around the tree, they hang ornaments on the branches . A
star or angle often crowns the top. Careful- wrapped gifts are placed beneath. Outside, families often string lights
around the windows and wind light around trees and shrubs in the front yard. As the families decorate their homes,
they often put on Christmas record. Almost every family has at least one favorite album or compact disc. School
children of all ages perform Christmas concerts for their parents and communities. On Christmas’ Eve, family
members gather around Christmas tree to sing traditional songs Such as Jingle bells and Silent night and then give
presents to each other.
Questions.
1- What are the popular traditions at Christmas?
a) Decorating the home. b) Singing Christmas carols.
c) Eating Christmas pudding. d) a and b are correct.
2- Where is the evergreen put ?
a)In the middle of the living room. c) In the middle of the bedroom.
b)In the corner of the living room. d) In the corner of the bedroom.
3- How do they decorate the Christmas tree?
a)They wrap string of colorful lights around the tree. c)They crown a star on the top
b) They hang ornaments on branches d) a,b and c are correct.
4- What do the family members often do on Christmas’ Eve ?
a) They gather around Christmas tree. b) They give presents to each other.
c) They sing traditional songs. d) They gather around the tree, sing traditional song and get presents
5- What do they do as they decorate their home ?
a) They sing traditional songs. c) They eat Christmas pudding.
b) They put on Christmas record. d) a, b and c are correct.
EX 16. Fill in each numbered blank with a suitable word
The country is (1).............beautiful than a town and more pleasant to live in. Many people think so, and go
to the country (2) .......... the summer holiday though they can't live (3) ............. all the year round. Some have
cottage built in a village (4) .......... that they can go there whenever they (5) ..........find the time.
English villages are not alike, but (6)............. some ways they are not very different from (7) ...................other.
Almost every village (8) ...........a church, the round or square tower of which can (9) ............seen from many miles
around. Surrounding the church is the church yard, (10).......... people are buried.
EX 17. Read the text then answer the question below.
Every child in Great Britain between the age of five and fifteen must attend school. There are three main types
of educational institutions: primary (elementary) schools, secondary schools and universities.
State schools are free, and attendance is compulsory. Morning school begins at nine o'clock and lasts until half
past four. School is open five days a week. On Saturdays and Sundays there are no lessons. There are holidays at
Christmas, Easter and in summer. In London as in all cities there are two grades of state schools for those who will
go to work at fifteen: primary schools for boys and girls between the ages of five and eleven, and secondary
schools for children from eleven to fifteen years.
The lessons are reading, writing, the English language, English literature, English history, geography, science,
Nature study, drawing, painting, singing, woodwork and drill (physical training).
1. What are three main types of educational institutions?
2. What are the two grades of state schools for those who will go to work at fifteen?
3. What subjects do children study at school?
4. Are state schools free?
5. When do pupils have holidays?
EX 18. Fill in each numbered blank with a suitable word
If you live in a city, you probably see many people, hear the (1)............of traffic, and smell the pollution
(2).........cars and factories. We are entering a new time in (3) ......history of the world. Before this, most (4) ..... .....
were farmers. They lived in the country. Now many people are (5) .....the farms and moving into the cities. They
are looking for better jobs. The cities are growing very quickly. Most cities are very crowded. People are driving
more cars, burning more fuel, (6).......more water, eating more food, making more garbage, and producing more
things in factories than (7)..........before. Life is becoming difficult. Some governments are trying to plan for the

5
future. They are building new roads, putting (8) .........new houses, looking for more water, and limiting growth in
certain areas. Still, city planners are getting worried. People are crowding into the cities (9) ....... than cities can
take them. The cities are running out (10).........room. What is the answer to this problem?
EX 19. Read the passage and choose the best answer.
It is very important to have healthy teeth. Good teeth help us to chew our food. They also help us to look
nice. How does a tooth go bad? The decay begins in a little crack in the enamel covering of the tooth. This
happens after germs and bits of food have collected there. Then the decay slowly spreads inside the tooth.
Eventually, poison goes into blood, and we may feel quite ill.
How can we keep our teeth healthy?. Firstly, we ought to visit our dentist twice a year. He can fill the small
holes in our teeth before they destroy the teeth. He can examine our teeth to check that they are growing in the
right way. Unfortunately, many people wait until they have toothache before they see a dentist.
Secondly, we should brush our teeth with a toothbrush and fluoride toothpaste at least twice a day- once after
breakfast and once before we go to bed. We can also use wooden toothpicks to clean between our teeth after a
meal. Thirdly, we should eat food that is good for our teeth and our body: milk, cheese, fish, brown bread,
potatoes, red rice, raw vegetables and fresh fruit. Chocolate, sweets, biscuits and cakes are bad, especially when
we eat them between meals. They are harmful because they stick to our teeth and cause decay.
1. Good teeth help us to ................
A. be nice B. have a good eyesight. C. chew our food D. be important
2. When food and germs collect in a small crack, our teeth ............
A. become hard B. begin to decay C. send poison into the blood. D. makes us feel quite ill.
3. A lot of people visit a dentist only when ..............
A. their teeth grow properly B. they have holes in their teeth
C. they have toothache D. they have brushed their teeth.
4. We ought to try clean our teeth ...............
A. once a day B. at least twice a day C. between meals D. before breakfast.
5. We shouldn’t eat a lot of ...............
A. red rice B. fresh fruit C. fish D. chocolate.
EX 20. Read the text then answer the questions below.
Fire was very important to man . He needed fire to keep himself warm at night. He used fire to cook his food. He
used fire to frighten away enemies and wild animals. In some parts of the world he used fire to signal messages.
Red Indians, for example, used fire to make smoke signals. In some other countries people lit fire to warm their
friends of danger. Fire was also used to give light. Before the invention of the oil lamp, men used burning sticks as
torches. And before man discovered gas and electricity, he hung small fires in wire baskets from posts to light the
streets. One man even used fire to tell the time. He invented a candle clock. He made a candle that took exactly
twelve hours to burn. Then he marked this candle in twelve equal parts. He lit the candle and could tell the time by
counting the number of parts left of the burning candle. But the candle clock did not always work well. If there
was a wind blowing on the candle, the flame burned too quickly.
1. What did man probably first use fire for?
2. For what purpose was fire used by red Indians?
3. What were the first street lights?
4. How long did the candle clock take to burn?
5. What would happen if a wind blew on a candle clock?
EX 21. Fill each of the numbered blanks in the following passage.
Food plays an ...1... part in the development of nations. In countries where food is ..2.., people have to
spend most of their time getting enough to eat. This usually slows down ..3.., because men have little time to
….4…. to science, industry, government, and art. In nations where food is ….5…. and easy to get, men have
more time to spend in activities that ….6…. to progress, and enjoyment of leisure. The problem of ……7….
good food for everybody has not yet been solved. Many wars have been ….8…. for food. But it is no longer
necessary to go to war for food . Nations are beginning to put scientific knowledge to work for a …9… of their
food problems. They work together in the Food and Agriculture Organization of the United Nations (FAO) to
help hungry nations …10… more food.

6
EX 22. Read the text and choose the correct word for each space:
ENGLISH AROUND THE WORLD
English is the first language of many people in countries outside the United Kingdom.
When you...1... speakers of English from around the world, you ...2.... notice that they do not all speak in the same
way. There are also some ......3...... in the words they use, including the names of ...4.... objects that are part of
everyone’s daily life. But although pronunciation and ...5... are not the same everywhere, it is interesting that
English speakers...6.... opposite sides of the world can understand ...7... other quite easily. It does not seem to ...8...
where they learnt the language. And of course this is one reason why speakers of other languages are keen ...9...
learning English too. If you know English, you are more...10... to be able to study or work in all sorts of exciting
places, such as the United States or Australia.
1. A- recognize B- meet C- find D- attend
2. A- originally B- strangely C- curiously D- immediately
3. A- mistakes B- corrections C- changes D- differences
4. A- common B- popular C- favorite D- general
5. A- reading B- composition C- dictation D- vocabulary
6. A- of B- in C- from D- at
7. A- each B- one C- the D- some
8. A- mine B- care C- matter D- worry
9. A- by B- on C- to D- for
10. A- likely B- probably C- possibly D- luckily

EX 23. Fill in each numbered blank with a suitable word


According to the writer, the modern father looks after his children and helps in the house, even
(1)............... his wife does not goes out to work. The division (2)................ the roles of the mother and the father is
no (3)............... very clear, and dad does (4).................. share of child care: he can change the baby, dress the
children or make the dinner. This new image of the father is, of (5)...................., completely (6) ..................from
the still traditional dad,(7).................... represents authority, is the head of the household (8).................makes all
the "important" decisions. His wife is (9).....................for the domestic side of family life while he
(10)..................the one who advises or punishes as necessary.
EX 24. Read the following passage and choose the best answer for each blank.
Chiang Mai, also sometimes written as “ Chiengmai”, is the largest and most culturally significant city (1)
……….. Northern Thailand, and the capital of the ChaingMai province. It is about 800 kilometers north of
Bangkok among some of the (2)………mountains in the country .The city stands on the ping river, a major
tributary of the chao phaya river. In recent years ChiangMai has become an (3)………modern city, although it
lacks the cosmopolitan gloss of Bangkok. It has many attractions for the thousand of foreign (4)…………who
come to the city each year. ChiangMai’s historic strength derived from its important strategic location, near a
southern branch of the ancient silk road, and long before the modern in flux of (5)……….visitor the city served as
an important centre for handcrafted goods, umbrellas, jewellery (particularly silver) and woodcarving.
Câu 1. A. on B.in C.of D.to
Câu 2. A. high B.short C.highest D.big
Câu 3. A. beautiful B.crease C.increasing D.to crease
Câu 4.A.visitors B.people C.they D.man
Câu 5.A. foreign B.tourist C.other D.area
* Choose the correct answer for each question
Câu 6. Where is the Chiang Mai?
A. in the east of the Thailand. B. in the south of the Thailand
C. in the north of the Thailand D. in the centre of the Thailand
Câu 7 .Which of the following sentences is true about Chiang Mai.
A.It is the largest city in Thailand.
B. It is the capital city of ChiangMai province.
C. It is nearly 500 kilometers from the capital of Thailand.
D. It isn’t surrounded high mountain
Câu 8.Chiang Mai is now…………………..
A.an ancient city B. famous for its vehicles

7
C.a quickly developing city D.near the silk road
Câu 9.Which jewellery is Chiang Mai most famous for?
A. umbrella B.silver C.woodcarving D.silks
Câu 10.What is word “significant” mean?
A.ancient B.large C.developed D.important
EX 25. Read the following passage and choose the best answer for each blank.
Tom, Mary, Ann and Judy are going to a birthdays party. Mary (1)……… a pink dress and a white hat and
gloves. Her (2)………are white and she’s wearing tights. She’s carrying a bag. Tom is wearing a black suit with a
white shirt and a blue tie .His shoes are (3)…………He’s carrying a coat .Ann is wearing a white shirt with a
jumper and trousers. His jumper is brown (4)……..his trousers are brown .He’s carrying a blue jacket . Judy is
wearing jeans and a yellow (5) ………..
Câu 1. A.is wearing B.wore C.wear D. worn
Câu 2. A.shirt B.shoes C.trouser D. umbrella.
Câu 3. A.a hat B.gloves C. coat D.black.
Câu 4. A.but B. or C.and D.so.
Câu 5. A.sweat shirt B.trousers C.dress D.jacket.
EX 26. Read the following passage and choose the best answer for each blank.
Halloween is a holiday. It is on the night of October 31 st . It is ………(1) in many English Speaking countries .
Children wear costumes. They go to people’s homes. The people give them ……….(2) candy. Children say “trick
or treat” to ask for candy. This comes from a treat .It means, “Give me a treat ……(3)I will play a trick on you”
Children today ……….(4) not do the tricks if they don’t get treats .But some children still do mischief playful
pranks or things to make fun of people like putting toilet paper in trees or writing …….(5) soap on windows .In
these countries Halloween is about ghosts, witches, goblins, and other scary things.
1 . A. presented B. proposed C.celebrated D.played
2 . A. candy B.cake C.meat D.egg
3 . A . and B.but C.or D.because
4 . A . never B.sometimes C. just D.usually
5 . A . for B.with C.often D. on to
* Choose the correct answer for each question.
Câu 6. Who is Halloween celebrated for?
A. Halloween is celebrated for children in English speaking countries.
B. Halloween is celebrated for adult.
C. Halloween is celebrated both adult and children.
D. Halloween is celebrated for many countries.
Câu 7 . When does they celebrate?
A.They celebrate on night in early October.
B. They celebrate on night in late October.
C. They were on night of September 31st.
D. They are celebrated as the same time of Tet holiday
Câu 8 .What do children say to ask for candy?
A.They say “trick or treat”
B. They said “trick or treat”
C. They were said “trick or treat”
D. They have said “trick or treat”
Câu 9 . What image of a child can you see from the Halloween?
A.We can see a ghost, witch, goblin and scary thing
B. We can see an old man with long moustache
C. We can see a beautiful fairy
D. We can see an animal.
Câu 10 . Is Halloween different from festival in Vietnam?
A.It is different
B. It is the same in Vietnam
C. It is funny
D. It is boring

8
EX 27. Read the following passage and choose the best answer for each blank.
The large movement of the earth under the water causes a very large and powerful Tsunami ……..(1) called the
Asian Tsunami in most of the world .It was called the Boxing day Tsunami in England, Australia, South Africa
…..(2) Canada because It happened on the holiday they call Boxing Day .The Tsunami caused a lot of damage to
countries such as Philippines ,Thailand, Indonesia and Srilanka.
Waves as high as 30 m killed many people and damaged or destroyed a lot of buildings and other property .
over 225000 people died or were not found …….(4) the Tsunami .The wave traveled as far away as south Africa
(800km) where as many as 8 people died because of……..(5) water caused by the wave . Because of how much
damage was caused and the number of people the earthquake affected, over $7 billion was donated to help the
survivors and rebuild the areas damaged.
1. A. is B.were C.are D.was
2 . A.but B.and C.so D.or
3 . A. Thailand B.The USA C.Vietnam D.Australia
4 . A.before B.during C.after D.since
5 . small B.high C.tall D.large
*Choose the correct answer for each question.
Câu 6 . Which of the following sentences is true?
A. Only in the Tsunami called Asian Tsunami.
B. Many people died because of the high waves.
C. The Tsunami didn’t cause a lot of damage to Indonesia.
D. A lot of money wasn’t raised to help people.
Câu 7 . Why was the Tsunami called the Boxing Day Tsunami in England?
A. Because it happened when people were boxing.
B. Because it happened when people were collecting boxes.
C. Because it happened on Boxing Day.
D. Because it destroyed a lot of boxes.
Câu 8 . How high were the waves?
A. nineteen meters. B. one hundred and twenty nine meters.
C. twenty meters D. thirty meters.
Câu 9 . What were some people in South Africa killed by?
A. Volcano B. high Tornadoes C.high water D. typhoon
Câu 10 . What is “survivors” mean?
A. people were deal B. people were live
C. People are left alive D.offices are destroyed
EX 28. Read the following passage and choose the best answer for each blank.
July 20th 1969 was an important day . Two American landed on the …….(1) They went in a Spaceship .It’s
name was……..(2) on July 20th the spaceship landed in the sea of tranquility. The first thing the men found was
that the moon is covered with dust .The dust is so thick that the men left their foot prints where they ………(3) .
there is no wind, no rain to wipe them off .They left their footprints that last forever .
The astronauts walked on the surface of the moon. They picked ……(4) some rocks and put them in the
………….(5).Then they put a flag on the ground, on July 21st Apollo II left the moon and returned to the earth.
Câu 1 . A.sun B. star C.moon D.planet.
Câu 2 . A . Apollo B. Ufos C.support D.prisoner
Câu 3 . A.go B. walked C.went D.gone
Câu 4 . A.of B. with C.off D. up
Câu 5 . A. car B. ship C.spaceship D.rock
*Choose the correct answer for each question.
Câu 6 .What is “Apollo” ?
A. It is a car. B. It is a spaceship.
C. It is a stars. D. It is a gift.
Câu 7 .What were the astronauts’ nationality?
A.They are Japanese. B.They are British
C.They are Australian D.They are American
Câu 8 . Which thing did they put on the moon?
A.It is rock B.It is a flag C.It put a ground D.It is a tree

9
Câu 9 . When did they return to the earth?
A.They returned to the earth on July 12th ,1969.
B. They returned to the earth on July 3 rd ,1969
C. They returned to the earth on July 24th ,1969
D. They returned to the earth on July 21st ,1969
Câu 10 .Who landed on the moon first?
A. The first thing that they saw on the moon is covered with dust.
B. The first thing which they saw on the moon is covered with rocks.
C. The first thing which they found on the moon is wind.
D. The first thing that they found on the moon is flag.
EX 29. Read the passage and tick the true (T) or false (F) statements.
Tropical rain forests are found in Amazon region of South America, Central America, Africa, and Southeast Asia.
Rain forests are very important to the world’s climate. The Amazon rainforest alone receives about 30 to 40
percents of the total rainfall on the earth and produces about the same percentage of the world’s oxygen. Some
scientists believe that the decreasing size of rain forests will affect the climate on the earth and make it
uncomfortable or even dangerous for life. Saving our rain forests is an international problem. One country or even
a few countries can not solve the problem alone. The nations of the world must work together to find a solution
before it is too late
1. Tropical rain forests are very important to the world’s climate.
2. They produce about 30 to 40 percents of the world’s oxygen.
3. Increasing size of rain forests will make our life more dangerous.
4. A country can solve the problem of saving rain forests.
5. Saving rainforest is national problem
EX 30. Read the text below then choose the right sentence a, b, c or d.
Do you like seasons?
The four seasons in Britain are very different. Read about what Daniel and Fiona think about winter, spring,
summer and autumn.
Daniel
Autumn is very beautiful time of the year. The leaves on the trees go brown, red and yellow and they go down. I
love to watch it! We play in the parks a lot but we wear warm clothes, because the days get cold. School starts
again in autumn, at the beginning of September. I don’t like school!
Summer is the time for me! School finishes in July and we can play in the parks all the time. Sometimes it is very
hot. People are happier and they talk with their friends and go for a walks more. The days are very long. In July,
for example, it gets dark only about 10 o’clock. It’s very difficult to go to sleep at night!
Fiona
It’s very cold in winter! Sometimes, we have a lot of snow and we can make snowmen. I don’t like winter because
we can’t go for a walks very much. We wear thick clothes and in our house it sometimes gets cold. When we
come home from school at 4 o’clock, it is already dark. I don’t like it at all! But there is one good thing about
winter - Christmas!
After the long winter, spring is great! There are new leaves on the trees and the birds start to sing. We can go for
long walks again and play in the park. The days are longer and it doesn’t get dark until about six o’clock. It’s
much warmer but it rains a lot and the days are very windy.
1. a- Daniel likes autumn because the trees are beautiful.
b- Daniel likes autumn because school starts in September.
c- Daniel doesn’t play in the park in autumn because the days are cold.
d- Daniel goes to school with great pleasure.
2. a- School in Britain finishes in May. b- Daniel doesn’t like to meet his friends in the park in summer.
c- People in Great Britain sometimes have hot summers. d- In July it is dark at nine.
3. a- Daniel always goes to sleep at eight in summer because the days are shorter than in winter.
b- It is always very hot in Britain in summer.
c- Daniel doesn’t like autumn at all !
d- In summer Daniel is happier than in autumn because he doesn’t go to school.
4. a- Fiona makes snowmen every day in winter. b- Fiona likes to go for walks with her friends.
c- Fiona doesn’t like winter because it is cold in winter. d- It never gets cold in Fiona’s house.
5. a- Winters are short in Britain. b- Fiona comes home in the afternoon.

10
c- Fiona doesn’t like winter at all! d- It never snows in Great Britain.
6. a- Fiona likes spring very much. b- Fiona likes dark winter days.
c- It gets dark at 5 in the afternoon in spring. d- In Great Britain it doesn’t rain much in spring.
EX 31. Read the following passage and choose the best answer
I had feared that my companion would talk, but it was soon plain that there was no such danger. Two days passed
during which we did not exchange a single word. He seemed, indeed, absolutely unaware of my presence. He
neither read nor wrote, but spent most of his time sitting at the table and looking out of the window across the
pleasant parkland that surrounded the house. He sometimes talked to himself and said things half under his breath.
He bit his nails and once he produced a penknife and dug holes in the furniture until one of the attendants took it
from him. I thought at first that perhaps he was mentally ill. During the second day I even began to feel a little
nervous of him. He was extremely large, both broad and tall, with very wide shoulders and enormous hands. His
huge head was usually sunk low between his shoulders. He had dark, rather untidy hair and a big shapeless mouth
which open very now and then. Once or twice he began singing to himself, but broke off abruptly (bất ngờ) on
each occasion - and this was the nearest he seemed to get to noticing my presence.
By the evening of the second day I was completely unable to go on with my work. Out of a mixture of
nervousness and curiosity, I sat, too looking out of my window and blowing my nose, and wondering how to set
about establishing the human contact which was by now becoming an absolute necessity. It ended of with my
asking him for his name. He had been introduced to me when he arrived, but I had paid no attention then. He
turned towards me a very gently pair of dark eyes and said his name: Huge Belfounder. He added:" I thought you
didn't want to talk." I said that I was not at all against talking, that I had just been rather busy with something when
he arrived, and I begged his pardon if I had appeared rude.
It seemed to me, even from the way he spoke, that he was not only mentally ill, but was highly
intelligent; and I began, almost automatically, to pack up my papers. I knew that from now on I
should do no more work. I was sharing a room with a person of the greatest fascination.
1. How did Huge spend the first two days? 3. Huge didn't talk to the writer at first because
A. He worked as if the writer was not there. A. He didn't realize the writer wished to.
B. He talked and sang to himself from time to B. He thought the writer was rude.
time. C. He was feeling ill.
C. He spent his time making holes in the furniture. D. He was too busy.
D. He kept annoying the attendants. 4. The writer's attitude to Huge changed from
2. On the second evening the writer A. fear to nervousness.
A. tried in vain to start a conversation. B. nervousness to interest.
B. was feeling bored. C. curiosity to nervousness.
C. began to fell frightened of his companion. D. nervousness to unfriendliness.
D. could not concentrate on his work.

EX 32. Read the passage and put one suitable word into each gap:
People usually complain that they never have enough time to accomplish tasks. The hours and (1) ................
seem to slip away before many planned chores get done. (2) .................... to time management experts, the main
reason for this is that most people fail to set priorities about what to do (3) ................. They get tied down by
trivial; time-consuming matters and never complete the important ones.
One simple solution often used by those at the top (4) ................. to keep lists of tasks to be accomplished
daily. These lists order jobs from most essential to (5) ................. essential and are checked regularly through the
day to access progress. Not only is this an effective way to imagine time, but also it serves to give individuals a
much-deserved sense (6) .................. satisfaction over their achievements. People (7) ................. do not keep
lists often face the end of the work with uncertainty over the significance of their accomplishments, which over
time can contribute to serious problems in mental and (8) .................. health.
EX 33. Read the following passage and then choose the word
“My home is in the air – I do an enormous amount of traveling. It is a fast life and (1) ______ of work, but I like it
and that is the only way (2) ______ me. Everything is tiring – music, traveling – but what can I do? I am not (3)
______ to complaining. It is hard to imagine now (4) ______ I will ever be very long in one place. My home town
is on the Caspian Sea. There is sea, wind, sun and (5) ______ (many tourists and hotels. I have my own flat with
four or five rooms, but I am seldom there. If I am there for a day or two I prefer to (6) ______ with my mother
and grandmother. They live in a small house, (7) ______ it is very comfortable and my mother cooks for me. I like
good, simple food.
11
I have no wife, no brothers or sisters and my father (8)______ when I was seven. He was an engineer and I don’t
(9)______ him very well. He liked music very much and wanted me to (10) ______ a musician.”
1. A. most B. full C. complete D. more
2. A. for B. to C. in D. by
3. A. wanted B. taken C. used D. known
4. A. and B. so C. while D. that
5. A. far B. too C. much D. more
6. A. stay B. go C. do D. spend
7. A. but B. since C. even D. which
8. A. killed B. gone C. passed D. died
9. A. know B. remember C. remind D. see
10. A. become B. turn C. develop D. grow
EX 34. Read the passage carefully and choose the correct answer:
In the early years of television, educational specialists believed that it would be very useful in teaching and
learning. Many schools have brought television sets, intending to use them effectively to improve the quality of
education; but actually they are rarely used properly in classrooms. Meanwhile, children spending the majority of
their out-of-school hours watching TV and their typical school days proceed as if television did not exist.
There are some explanations for the failure of television to get the interest of the teachers. Firstly, the schools
that purchased television sets have not set aside money for equipment repairs and maintenance so these television
sets are sooner and later out of work. Secondly these schools have not found an effective way to train teachers to
integrate television into their ongoing instructional programs. Lastly, most teachers do not regard the quality of
television and its usefulness in the classroom.
Teachers at the schools work hard for at least twelve years to train their students to become good readers.
However, according to a recent statistics, teenagers seldom spend their free time reading books and newspapers
but watching television instead.
1) The text is about:
A. the use of television at schools. B. teaching and learning television
C. educational specialists. D. watching TV outside school.
2) When TV first appeared, educational specialists . . . . . . . .
A. did not appreciate it.
B. did not appreciate it.
C. believed it would be useful for schooling.
D. banned children from watching TV.
3) According to the text, TV . . . . . . . . .
A. has not been used properly in classrooms. B. has been used effectively in classrooms.
C. has not existed in classrooms. D. has not attracted students’ interest.
4) There are . . . . . . . . . explanations for the failure of television to get the interest of the teachers.
A. two B. three C. four D. Fiv
5) Children spend their free time . . . . . . . . . . . . . .
A. reading books B. reading newspapers.
C. learning foreign languages D. watching TV.
EX 35. Fill in each blank with one suitable word.
The Browns live in the countryside. Last week when they came up to London, they …1… their times on
seeing as much as …2… . Mrs Brown was most interested in stores, especially in Oxford Street, and bought some
of …3… things which she could not find in the shops in the countryside. Their two children, George and Susan,
…4… had never been to London before, were surprised at the crowds everywhere. They enjoyed travelling on the
underground and going down the moving stairs …5… lead to the platforms.
One day they went to Hyde Park and walked along on the grass to Kensington Garden in the bright autunm
sunshine. It was very quiet here. Only the noise from the streets reminded them that they were still in a city. The
…6… of the tall trees were turning red, brown and yellow, birds were …7… about, sheep were eating the grass,
there were …8… of flowers, and ducks were swimming about on the Round Pond. George, …9… hobby was
planes and boats, was very much interested in the model boats which boys were sailing …10… the Pond.
Before the end of the week they had seen a great. They hoped, however, to see more of London on their
next visit.

12
EX 36. Read the text below then choose the right sentence
BABY
It was a beautiful spring day: the sun was shining, the sky was blue. In the centre of London a policeman cried. He
saw a man with a big lion. They were walking down the street.
“Hey,you!” he said. “What are you doing here with this lion? You can’t walk around the streets with a lion.
Take it to the Zoo!”
“OK, officer. I want to show Baby the town.”
The man opened the door of his car and the lion jumped in. The car went away.
The next day the police officer saw the same man and the same lion again.
“Hey, you!” he said.”Come over here! And bring that lion with you!”
The man took the lion to the police officer.
“What’s the problem, officer?”
“Problem? I told you yesterday to take the lion to the Zoo!”
“Oh, I did, officer, I took Baby to the Zoo. He enjoyed it very much. But today, I am taking him to the
swimming pool !”
1. a- It wasn’t raining that day.
b- A policeman saw a man with a dog in the centre of New York.
c- The man and his pet were walking along the park.
d- The man didn’t have a car.
2. a- The lion couldn’t get into the car, the lion was too big.
b- The policeman took the lion to the Zoo and put the lion into the cage.
c- The man showed his pet the Zoo.
d- The policeman was happy to see a man with a lion in the centre of London.
3. a- The man had a baby. It was a nice girl of three.
b- Baby was the lion’s name.
c- The policeman told the man to show Baby the town.
d- The lion visited the London Zoo.
4. a- The man could drive a car.
b- The lion didn’t like the Zoo at all.
c- The policeman took the lion to the swimming pool.
d. The policeman met people with lions in London streets every day.
5. a- The lion was the man’s pet.
b- The man had a baby lion as a pet.
c- The policeman showed the park and the school to the lion.
d- When the policeman saw the man with the lion he got very hungry.
EX 37. Choose the suitable word in the box to fill in each blank to complete the passage.
foreign give hear in
languages meaning newspapers useful
Students of foreign (1)………..need a dictionary which contains all the words in common use (2) ……………
their own language and the one they are trying to learn, that is, the words they are likely to (3)………….. in
conversations and on radio, and those they will meet in the books and (4)…………they read. Such dictionaries
usually give the (5)……………….of a word by translating it; and sometimes, but not always, they (6)…………
translations of phrases and structures. Dictionaries of this kind are useful to translators, but less (7)……….to
earnest students of language than dictionaries which give meanings, explanations and examples in (8)……………
language itself.

13
EX 38. WHERE DO WE GET THE NEWS?
Information about what has happened in the world, in our country, in our area, or even in our town comes
to us in many ways. People who can read often get their news from media. Because many people in the world are
illiterate, however, they get the news from their television or radio.
Newspapers around the world are similar in many ways. They all have news stories that tell people about
the event of the day or week. Often, papers have other purposes that are devoted to business, sports and maybe the
latest fashion news. Papers usually appear weekly or everyday, but in some places they may only appear monthly.
Magazines are another way that some people get the news. They are longer and usually appear slower. Television
and radio are two other ways of the news. They get the news out faster than the newspapers and magazines, and
they don't have to be read. Radio and television stations broadcast the news several times each day.
1. Where do people who can read get the news?
2. How do the illiterate people get the news?
3. What are the purposes of newspapers around the world?
4. Which is quicker? A newspaper or a magazine?
5. Do magazine and newspaper have their broadcasting stations?
6. How many means of media are mentioned in the text? What are they?
EX 39. Complete the passage with one word
The Eiffel Tower . . . (1). . . . .built for the International Exhibition of Paris celebrating the centenary of
the French Revolution. The Prince of Wales, later King Edward VII of . . (2). . . . . ., opened the tower. Of the 700
proposals submitted in a design competition, Gustave Eiffel's was unanimously chosen. However it was not
accepted . . . .(3). . . . . all at first, and a petition of 300 . . . .(4) . . . . including those of Maupassant, Emile Zola,
Charles Garnier and Dumas the Younger - protested its construction.
. . . (5). . . 320.75 meters including antenna, and 7000 tons, it was the world's tallest
. . (6). . . . . . until 1930. Other statistics include: 2.5 million rivets. 300 steel workers, and 2 years (1887-1889) to
construct it. Sway of at most 12 cm in high winds. Height varies up to 15 cm depending . . . (7). . . . temperature.
15,000 iron pieces (excluding rivets). 40 tons of paint 1652 steps . . . (8). . . . the top.
It was almost torn down in 1909, but was saved because . . .(9) . . . . . its antenna - used for telegraphy at
that time. Beginning in 1910 it became . . . .(10) . . . .of the International Time Service. French radio and French
television . . .(11) . . . . . also made use of its stature.
During its life-time, the Eiffel Tower has also witnessed a . . .(12) . . . . .strange scenes, including being
scaled by a mountaineer in 1954, and parachuted off of in 1984 by two Englishmen. In 1923 a journalist . . . .(13) .
a bicycle down from the first level. The tower has three platforms. An. . . (14). . . restaurant, the Jules Verne, is on
the second platform. The top platform . . . .(15) . . . a bar, a souvenir shop and the office of Gustave Eiffel.
EX 40. Read the text and decide if the following statements are true (T) or false (F)
Almost a hundred thousand people were killed and half a million homes destroyed as a result of an
earthquake in Tokyo in 1923. The earthquake began a minute before noon when people were cooking their midday
meals. Thousands of stoves were overturned as soon as the earthquake began to shake. As a result, small fires
broke out everywhere and quickly spread. The fire engines were prevented from going to help because many of
the roads had cracked open. It is impossible to use fire fighting equipments. Consequently, over ninety percent of
the damage was caused by fire rather than by the collapse of buildings.
1. The earthquake began a minute before midnight.
2. People used fire fighting equipments to prevent fires.
3. A lot of people were killed in 1923 because of a fire.
4. Small fires broke out everywhere because of the overturned stoves.
5. Over ninety percent of the damage was caused by fire.
EX 41. Complete the passage with one word
The Pilgrims left their home in England in (1) ……… of religious liberty. After a long, hard (2) …………..
across the Atlantic, their ship, the Mayflower, finally (3) ………….. land. In November, 1620, the Pilgrims
sailed into Cape Cod Bay in Massachusetts to start (4) ……. new life. The first winter was (5) ………… of
hardship. Then in the spring they planted seeds, and all summer long they worked (6) ………… their farms and
prayed for good crops. When fall came, they had a very good harvest with plenty of food for the winter. In
addition, the men went (7) …………. in the woods and shot wild turkeys. The Pilgrims were very thankful.
They prepared a great feast and invited their friendly Indian (8)………… to enjoy it with them.
In memory of that happy day Americans (9) ……… Thanksgiving Day every year. They (10) ….. relatives and
friends to eat turkey and other tasty foods and to give thanks for all good things.

14
1. a. interest b. look c. search d. need
2. a. trip b. voyage c. journey d. travel
3. a. came b. went c. reached d. got
4. a. his b. her c. their d. its
5. a. filled b. full c. covered d. lack
6. a. at b. in c. for d. on
7. a. fishing b. hunting c. catching d. searching
8. a. people b. neighbors c. residents d. settlers
9. a. hold b. organize c. celebrate d. give
10. a. ask b. call c. want d. invite

15
EX 42. Complete the passage with one word
 According to a group called The Voices Foundation, everyone has a singing voice as well as a speaking voice
somewhere inside them . This, they say, should be encouraged from an early (1) ............... …… ……..because it
provides the best, and the cheapest, (2)............... ……. ….. on which to build an understanding of music.
 (3) .................the Foundation ideas, lies the teaching of the Hungarian composer Zoltan Kodaly, He observed
that song can (4)............... as key part of the relationship between mother and child almost from birth. This is
especially(5) ............... of more traditional societics, like those of West Africa, where some small children are (6)
..................to sing literally hundesds of songs, all of which have been learnt by (7) ................ But many modern
children first (8) ............... to an understanding of music when they learn to play an instrument, and (9) ...............
some teaching of the theory of music is usually a part of thix, their relationship with the music on
the(10) ................. is often a mechanical one .
The(11) ............... of the Voices Foundation is that a natural (12) ............... for rhythm, harmony and musical
structure, the very(13) ............... we appreciate in the greatest musicians, can only be achieved through the
exploration of music with the voice from the start . The foundation has, therefore, (14) ............... itself  the task of
developing a singing -cectred musical education progamme that could(15) ............... junior pupils all over the
world.
EX 43. Seven of the eight sections below are part of a story but they are in the wrong order. One
section does not belong to the story, leaving out that one. Put the sections in the correct order and
write a complete passage.
A. It was evening on Tuesday, March 1st, 1932. Charles and Ann Lindbergh put their son Charles Junior to bed
and had dinner together, after which Lindbergh went to work in his study room.
B. The note was not signed. Instead, at the bottom of the page, there were two open blue circles and a filled blue
circle where they touched.
C. Lindbergh picked it up but did not open it. Instead, he called the police.
D. The police searched his rooms again and this time they found a diary with a list of the names of people who
bought and sold paintings in America, France and Germany.
E. When they arrived they opened the envelope and found a sheet of paper with this note written on it.
F. At about 9 o'clock, he heard a noise like something breaking, but decided it was thunder. At 10, Mrs. Lindbergh
went to check that Junior was asleep. The bed was empty and the child was missing. She ran downstairs to get
her husband.
G. “Have $50,000 ready in small bills. In two days, we will tell you where to deliver the money. Do not inform the
police. The child is in good care.”
H. They ran into the child's bedroom and stared at the empty cot in horror. Lindbergh looked around and saw that
the window was open. On the floor in front of it was an envelope.
EX 44. Choose the best answer A, B, C or D to complete the passage.
Society has changed in many ways (1)_____ the introduction of computers, and people's lives at home and at
the office have been affected. Most people are working for fewer hours per week than they (2)_____ to.
Manufacturers and advertising agencies are becoming much more interested in how people spend this extra
leisure time. One recent report stated that, (3)_____ the number of hobbies had not increased, each hobby had
become more specialized. A second finding is that nowadays, many managers would rather (4)_____ time
with their families than stay late in the office every day. Home life is seen to be just as important as working.
Some companies now (5)_____ managers take their annual holidays even if they don't want to, because this
leads to such an improvement in their performance if they have some rest. (6)______ these changes, some
people are working harder than ever before. The standard of exams is getting higher, and increased
competition is (7)______ it harder to get into university than it was 20 years (76)_____. School children and
students are now having to work (8)______ hard that in many cases they work longer hours (9)_____ their
parents.
1) from B. since C. for D. at
2) A. have B. want C. ought D. used
3) A. because of B. as C. although D. but
4) A. spend B. to spend C. spending D. spent
5) A. force B. make C. cause D. have
6) A. In spite of B. Despite of C. Although D. Because
7) A. doing B. making C. causing D. working
8) A. later B. after C. ago D. now

16
9) A. very B. such C. too D. so
10) A. then B. than C. as D. compare
EX 45. Choose the best option to fill in each of the blanks to make a meaningful passage
Most people think of computers as very modern inventions, products of our new technological age. But actually
the idea for a computer was (1) ______ out over two centuries ago by a man (2) ______ Charles Babbage.
Babbage was born in 1791 and (3) ______ up to be a brilliant mathematician. He drew up plans for several
calculating machines (4)______ he called engines. But despite the fact that he (5) ______ building some of these
he never finished any of them. Over the years people have argued (6) ______ his machines would ever work.
Recently, however, the Science Museum in London has finished building (7) ______ engine (8) ______ on one of
Babbage's designs.
It has taken six years to complete and more (9)______four thousand parts have been specially made. Whether it
works or not, the machine will be on show at a special exhibition in the Science Museum to (10)______ people of
Babbage's work.
  1. a) turned b) thought c) invented d) worked
2.  a) known b) recognized c) called d) written
3. a) developed b) grew c) brought d) expected
4. a) which b) who c) there d) whose
5. a) wanted b) made c) started d) missed
6. a) until b) whether c) while d) though
7. a) an b) the c) some d) that
8. a) depended b) based c) insisted d) influenced
9. a) than b) therefore c) when d) then
10. a) remind b) say c) inform d) encourage
EX 46. Choose the word or phrase that best fits the blank space in the following passage
Mr. Brown and some (3) __________________ conservationists are on a very dirty beach now. Today they are
ready to make the beach a clean and beautiful place again. After listening to Mr. Brown’s instructions, they are
divided (4) ____________ three groups. Group I needs to walk along the shore. Group 2 should check the sand,
(5) _________ group 3 has to check among the rocks. Garbage must be put into plastic bags, and the bags will be
(6) __________ by Mr. Jones. He will take the bags to the garbage (7) _________. Each member will be given a
map to find the right place. They won’t eat the picnic lunch (8) __________ by Mrs. Smith until the whole area is
clean. (9) _________ are eager to work hard so as to refresh this (10) ________ area.
3) a) voluntary  b) volunteers  c) volunteering  d) volunteer
4) a) in  b) to  c) into  d) onto
5)  a) or  b) and  c) because  d) though
6) a) selected  b) chosen  c) collected  d) elected
7) a) dump  b) yard  c) area  d) place
8) a) happened  b) provided  c) achieved  d) shown
9) a) Them all  b) They all c) All them  d) All they
10) a) spoiling  b) spoil  c) spoiled  d) spoils

EX 47. Fill in each numbered blank with ONE suitable word to complete the passage.
Some hundred years ago there was no equality between men and women because people then (1) __________
women to be the weaker sex. This prejudice against women had its origin in the dawn of mankind's (2) ________
when men went hunting for food. The task of food gathering and hunting needed great (3) _____________ of
body. Therefore, the best place for women was not in the forest, but at home where they could satisfactorily do
their job, namely (4) ____________ their children and looking after them. Things have changed much since those
early days. In the modern life of the 20th century, more (5) ______________ are needed not more strength of the
muscle. It's a consequence that women play an increasingly important role in the (6) ___________ society. They
have proved repeatedly they are equal and often superior to men in almost every field.

EX 48. Choose the word with the different tress patterns.


1. A planet B exist C routine D behind
2. A encourage B primary C inventor D addictive
3. A competitor B requirement C temperature D mysterious
4. A annoyance B government C experiment D participate

17
5. A representative B satisfactory C documentary D deforestation
EX 49. Write the correct form if the words
1 . Many________________ (CULTURE) activities will be held tomorrow.
2. We couldn’t win the prize. We were ____ ( SUCCESSFUL).
3. Our team played________________ ( SUCCESSFUL). We lost.
4. This man is ________________ ( SUCCESS) in writing.
5. The manager sacked five ________________ ( EMPLOY).
6. The author’s ________________ (HAPPY) is communicated through his song.
7. We find Huong very________________ ( COMMUNICATE).
8.He came first in the ______________ ( COMPETE). The other ______________ ( COMPETE) admired him.
9 . The ________________ ( ATTRACT) of the moon for the earth causes the tides.
10. " Back street Boys" is greeted________________ ( WARM) in the US nowadays.
EX 50. Read the passage and choose the correct answer
THE FIRST WOMEN SCIENTIST
Hypatia was born in Alexandria, on 370 A.D. For many centuries she was (1)....only woman scientist to have a
place in the history books. Hypatia’s father was director of Alexandria university, and he (2).... sure his daughter
had the best education available. This was unusual, as most women then had few (3)....to study. After studying
in Athens and Rome, Hypatia returned to Alexandria (4)....she began teaching mathematics. She soon became
famous (5).....her knowledge of new ideas.
We have no copies of her books ,(6)....we know that she wrote several important mathematical works. Hypatia
wa also interested in technology and (7).....several scientific tools to help with her work . At the (8)......many rules
were afraid of science, and (9)....connected with it was in danger. One day in March, 415, Hypatia (10)....attacked
in the street and killed.
1. A one B the C a D an
2. A could B made C said D put
3. A classes B customs C opportunities D teachers
4. A where B how C there D which
5. A from B by C for D in
6. A because B but C or D as
7. A did B experimented C invented D learned
8. A day B period C year D time
9. A anyone B nobody C all D something
10.A was B had C has D is
EX 51. Read the passage then choose the best answer to the questions
I often hear or read about "natural disasters" - the eruption of Mount St. Helen, a volcano in the state of
Washington; Hurricane Andrew in Florida; the floods in the American Midwest; terrible earthquakes all over the
world; huge fires; and so on and so on. But I'll never forget my first personal experience with the strangeness of
nature - "the London Killer Fog" of 1952. It began on Thursday, December 4, when a high-pressure system (warm
air) covered southern England. With the freezing-cold air below, heavy fog formed. Pollution from factories, cars,
and coal stoves mixed with the fog. The humidity was terribly high, there was no breeze at all. Traffic (cars, trains,
and boats) stopped. People couldn't see, and some walked onto the railroad tracks or into the river. It was hard to
breathe, and many people got sick. Finally on Tuesday, December 9, the wind came and the fog went away. But
after that , even more people got sick. Many of them died.
1. Which "natural disaster" isn't mentioned in the text?
A. a volcano B. a flood C. a hurricane D. a tornado
2. What is his unforgettable personal experience?
A. the London killer B. the heavy fog in London in 1952
C. the strangeness of natural D. a high-pressure system
3. How long did the "London Killer Fog" last?
A. for four days B. for five days C. for six days D. for a week
4. What didn't happen during the time of the "London Killer Fog"?
A. pollution B. humidity C. heavy rain D. heavy fog
5. Why did the traffic stop?
A. Because of the rain B. Because of the windy weather
C. Because of the humid weather D. Because of the heavy fog

18
EX 52. Fill in each blank with one suitable word
New Year is one of the four important traditional (1).............in the United States . On New Year’s Eve , most
people go to parties. At twelve o’clock (2).......night, everyone says “Happy new Year” and they (3)............ their
friends and relatives good luck. New Year’s parties usually (4)........a long time. Some people don’t go home
(5)...........morning. Another holiday, Halloween , is mainly for children . On this holiday children dress
(6).......witches, ghosts or others . Most of the children go (7).............house to houses and say “Trick or Treat”.
(8)........the people at the house do not give them candy, the children will (9)..............a trick on them. But this
(10)............ever happens. Almost people give them candy or fruit.
EX 53. Choose the best reply from B for each question in A
A Answer B
1. Do you like dancing? A. I’m an engineer.
2. Would you like to dance? B. Yes. Do you know a good restaurant?
3. Would you like a Chinese meal tonight? C. Half a million VND.
4. How old were you when you started school? D. It’s rainy.
5. What do you do? E. Yes, especially modern.
6. How are you ? F. I’m Vietnamese.
7. What’s the weather like in Hanoi now? G. Six years old.
8. What’s your nationality? H. Twenty minutes.
9. How much does it cost? I. Thank you ,but I’m too tired now.
10.How long does it take to get there? J. Fine, thanks

EX 54. Choose the best answer (A, B, C or D) for each space


Jeans are very popular with young people all (1)……. the world. Some people say that jeans are the
“uniform” of (2)…….. But they haven’t always been popular. The story of jeans started (3)……… two hundred
years ago. People in Genoe, Italy made pants. The cloth (4)…. in Genoa was called “Jeans”. The pants were called
“Jeans”. In 1850, a saleman (5)….California began selling pants made of cancas. His name was Levi Strauss. (6)
….they were so strong, “Levi’s pants” became (7)…… with gold miners, farmer and cowboys. Six years (8)…..,
Levis began making his pants with blue cotton (9)……called denim. Soon after, factory workers in the United
States and Europe began (10) …..jeans. Young people usually didn’t wear them.
1. A. in B. on C. over D. above
2. A. youth B. young C. younger D. youngest
3. A. lots B. much C. most D. almost
4. A. make B. makes C. making D. made
5. A. in B. on C. at D. with
6. A. Although B. But C. Because D. So
7. A. famous B. popular C. good D. wonderful
8. A. late B. later C. latest D. last
9. A. cloth B. clothing C. cloths D. clothes
10. A. wear B. wearing C. wore D. worn
EX 55. Fill in the gaps with one suitable word in each gap in the following passage.
Learning a foreign language is, in some ways, like learning how to fly or (1)__________ the piano. There are
important differences, but there (2)__________ a very important similarity. It is this: learning how to do such
things needs lots (3)__________ practice. It is never enough simply to “know” something. You must be able
to “do” things with what (4)__________ know. For example, it is not enough simply to read a book on
(5)__________ to fly an aero plane. A book can give you (6)__________ of information about how to fly, but
you only read a book and then try to (7)__________ without a great deal of practice first, you will crash and
kill (8)__________. The same is true of (9)__________ the piano. So you think it is enough simply to read
about it? Can you play the piano without having lots of (1)__________ first?
EX 56. Read the following passage and then answer the questions below.
Greece today is a small country in southeastern Europe. The population is approximately nine million, and
the capital city is Athens. Hundreds of islands surround the mainland. The largest island is Crete. No part of
the nation is far from water. The Lonian Sea and the Aegean Sea are famous for the long coastline. Greece is
very well-known for its shipping industry. More than three thousand years ago, the Greek people developed a
very complicated society. They had a great civilization, one of the greatest that the Western world has ever
19
seen. Greek architecture, thinking, and art influenced other civilizations. Even the Greek language influenced
other languages, including English. For example, the English words alphabet, democracy, and arithmetic
come from Greek. Today Greece is one of the most popular nations with the tourists who visit Europe.
Thousands of people are attracted to the country because of its beautiful scenery, magnificent ancient
buildings, and its excellent summer weather.
Questions:
1. Where is Greece? .............................................................................................................
2. Is it far from the main land to the coastline?......................................................................................
3. In what fields has Greece influenced other civilizations?....................................................................
4. Why does Greece attract many tourists?.....................................................................................
5. When did the Greek civilization begin?............................................................................................
EX 57. Fill in each of the blanks with one suitable word to complete the following passage:

are the of has most and after with each who

ENGLISH UNIVERSITIES AND COLLEGES


All English universities except Oxford and Cambridge (1) _______________ fairly new. London
University is (2) _______________ biggest of modern English Universities and consist of a great variety
(3)_______________ colleges and other institutions including medical schools.
A university usually (4) _______________ faculties(các khoa) and departments. The(5)___________
common faculties are medicine, law, arts (6) _______________ science. The departments include
engineering, economics, etc. (7) _______________ taking examinations, a university graduate is awarded (8)
_______________ the Degree of Bachelor of Arts, Science, Engineering, etc .
(9) _______________ faculty is headed by one or more professors, (10) _______________ are helped by
a staff of teachers called lecturers.
EX 58. Fill in each blank with the best word: many, about, look, colorful, cloth, other, …
Clothes can tell a lot ………………………………a person. Some people like very colorful clothes because
they want everyone to……………………………at them and they want to be the center of
things………………………..people like to wear nice clothes, but their clothes are
not………………………………or fancy. They do not like people ………………………………….Clothes
today are very different ……………………………..the clothes of the 18oos. One different is the way they
look. For example, in the 1800s, all women………………………..dress. The dresses had long skirts. But
today, women do not always wear dresses with long skirts. Sometimes they wear short skirts, sometimes they
wear pants. Another difference between 1800s and today is the ………………………….. In 1800s, clothes
were…………………………natural kinds of cloth. They were made from cotton, wood, silk or linen. But
today, there are………………………….kinds of man made cloth. A lot of clothes are now made from nylon,
rayon or polyester.
EX 59. Fill in each blank with the best word:
A story is a work of imagination. The people (1)……….write stories write them in order to give pleasure to (2)…………….
who read stories. Story-readers are, generally(3)……………., women of all ages and young men. Readers love the start
of a story, where there are new and sometimes strange people to be (4)……………for the 1st time. They enjoy the story
itself, the gentleness and the violence, the loves and the (5)…….………, with which a good writer interests his (6)……… .
They enjoy the end of the story, whether it is happy or (7)…… . The reader’s chief purpose in all this is to (8)
(9) (10)
………..from ordinary life for a short ……….. . Older men, as a rule, find their ordinary lives ……………….pleasant to
run away from.
EX 60. Read the passage below and do the task that follows.
Learning another language gives the learner the _____(1) to step inside the mind and context of that other culture.
Without the ability to communicate and understand a culture on its own terms, true access _____(2) that culture is
barred. Why is this important? In a world where nations and people are ever more dependent on another to supply
goods and services, solve political disputes, and ensure international security; _____(3) other culture is paramount.
Lack of intercultural sensitivity can lead to _____(4) and misunderstandings, to an inability to cooperate,
negotiate, and compromise, and perhaps even to military confrontation. Intercultural understanding begins with
individuals who _____(5) language abilities and who can thereby provide one’s own nation or community
_____(6) an insider’s view into foreign cultures, who can understand foreign new sources, and give insights into
other perspectives on international situations and current events. For survival in the global community, every

20
nation needs such individuals. A person competent is other languages can bridge the gap between _____(7),
contribute to international diplomacy, promote national security and world peace, and successfully engage in
_____(8) trade.
EX 61. Read the passage below and do the task that follows.
June the first is Pen Pal Day. I’m sure most of us have had a pen pal at one time _____(1) our lives. Many of
us still have one. That is the day to say thank you to your pen pals. It is also the day to thank your key pals,
too. Nowadays, most people _____(2) via e-mail, chat or text messaging. We type using keys, rather _____(3)
writing with pens. Using the latest technology, we can also have webcam pals. Times have changed _____(4)
the original ideas of a pen pal. Many language students have pen pals because it’s a great way to practice
writing in another language. There are many websites people can visit to find a pen pal. If you would like a
pen pal, perhaps this is the way to start looking for and _____(5) a new friend.
Being pen pals can lead to lifelong friendships. Some pen pals arrange to meet face to face. It is interesting to
communicate _____(6) other people and share our hobbies, interests and facts about our life. Many of us are
pen pals without even knowing it. Perhaps we all strike up a friendship with some one online and then keep
_____(7) contact. It might be that the Internet will change the idea of a pen pal forever. The _____(8) mailing
of a letter written on paper and then mailed in an envelope may soon disappear. Many traditionalists say it is
more exciting to receive a real letter through the post. Before the Internet, opening and reading a letter form a
pen pal was special.
EX 62. Read the passage below and do the task that follows.
One of the first novels in the history of literature ______(1) Robinson Crusoe, which Daniel Defoe, the Great
English _____(2), wrote in 1718 when he was nearly sixty _____(3) old. Daniel Defoe was born _____(4)
London in the family of a rich man. When he was still a school _____(5), he began to write stories. After he
_____(6) school, he worked at his father's shop. While he was _____(7) there, he continued to write stories
_____(8) a newspaper. Defoe _____(9) many countries and met a lot of people (10) _____ helped him much
in his writing.
EX 63. Read and answer:
nce there was a young Inca boy. He had no family except for an old llama. Each day the boy and his llama
walked many miles, looking for a home. Each night they curled up together and slept. But one starry evening,
the old llama died. The boy buried his friend next to an icy stream. Then he sat under a tree and cried. What
would he do? He had no family and no home.
The boy cried for a very long time. But there was no one to comfort him. There were only the stars in the sky.
Suddenly, the sky filled with bright light. The boy held his breath. He was afraid to move. One bright star fell
to the ground. Slowly, the star took the shape of the old llama. She bent her head and drank from the stream.
She looked at the boy and smiled. As she jumped back into the sky, bits of llama wool fell.
As the Sun began to rise, the boy picked up the soft, warm wool. It glowed in his hands like starlight. He
carried the wool to the city and sold it. With the money, he bought a house. He bought two young llamas. He
never forgot the star llama. And he was never lonely again.
1) Why does the boy cry when the old llama dies?
A) because he is afraid of the dark
B) because he is always sad
C) because the old llama is his only family
D) because he is lost
2) What happened BEFORE the old llama died?
A) The boy bought two new llamas.
B) The boy and the llama walked many miles looking for a home.
C) The boy collected the wool and sold it in the city.
D) The old llama fell to the ground in the shape of a star.
3) Read this sentence from the story.
One bright star fell to the ground.
What does bright mean?
A) old
B) shining
C) new
D) clean
4) What does the boy in the story want?
21
A) a cart
B) a horse
C) a new wool shirt
D) a home
5) What happens at the end of the story?
A) The boy buys a house and two llamas.
B) The boy buries the llama next to an icy stream.
C) The boy and his llama walk for many miles.
D) The boy cries for a very long time.
6) Read this sentence from the story.
As the Sun began to rise, the boy picked up the soft, warm wool.
What does rise mean?
A) grow darker
B) fall down
C) come up
D) take a step
EX 64. READ AND ANSWER:
D o you know how blueberries grow? They grow on bushes. Each blueberry is small and round.
Many blueberries can grow on one bush. At first, the blueberries are green. The green berries are
not ready to eat yet. They need a lot of sun and rain to help them become fat and sweet. When the
berries turn blue, they are ripe and ready to be picked.
Some farmers grow blueberries in big fields. The people who live nearby can earn money by
helping to pick the blueberries. Each one takes a pail out to the field and fills it with blueberries.
They work fast so that they can fill many pails. They want to earn as much money as they can.
When they are done picking, their fingers are blue from the juice of the berries!
After the blueberries are picked, they are put into boxes and sent to stores. People buy the
blueberries and take them home to eat. Some people like to wash the berries and eat them one by
one. Other people like to cook with blueberries. They make blueberry muffins and pancakes.
No matter how you eat them, blueberries taste great!
1) Read this sentence from the story.
The people who live nearby can earn money by helping to pick the blueberries
What does earn mean?
A) sell
B) get
C) lose
D) share
2) What color are the blueberries BEFORE they are ready to be picked?
A) blue
B) red
C) green
D) brown
3) Where do the blueberry pickers work?
A) in a barn
B) in a field
C) in a forest
D) near a stream
4) What happens to blueberries AFTER they are picked?
A) They are put into boxes.
B) They need sun and rain.
C) Farmers plant them.
D) They become sweet and ripe.
5) Read this sentence from the story.
They need a lot of sun and rain to help them become fat and sweet.
What does sweet mean?

22
A) juicy
B) sour tasting
C) dry
D) sugary tasting
6) What could be another title for this story?
A) "All About Blueberries"
B) "How to Cook with Blueberries"
C) "Eating Blueberries"
D) "All About Strawberries"
EX 65. READ AND ANSWER:   JULIE’S RACE
The dogsled race was about to begin. Julie’s team of dogs was lined up at the starting gate. Julie
stood behind them. The air was so cold that she could see her breath. Other teams were lined up,
too, and the dogs were excited. Julie kept her eyes on the clock. At exactly ten o’clock, she and
the other racers yelled, "Mush!" The dogs knew that meant "Go!" They leapt forward and the race
began!
Julie had trained months for this race, and she hoped she and her dogs would win. Hour after hour,
day after day, Julie’s dogs pulled the sled in order to get in shape for the race.
Now, they ran over snowy hills and down into frozen valleys. They stopped only to rest and eat.
They wanted to stay ahead of the other teams. The racers had to go a thousand miles across
Alaska. Alaska is one of the coldest places on Earth. The dogs’ thick fur coats helped keep them
warm in the cold wind and weather. In many places along the route, the snow was deep. Pieces of
ice were as sharp as a knife. The ice could cut the dogs’ feet. To keep that from happening, Julie
had put special booties on their feet.At first, the dogs seemed to pull the sled very slowly. They
were still getting used to the race. But on the third day out, they began to pull more quickly. They
worked as a team and passed many of the other racers. Once, one of the sled’s runners slid into a
hole and broke. Julie could have given up then, but she didn't. She fixed it and they kept going.
When they finally reached the finish line, they found out that they had come in first place! It was a
great day for Julie and her dogs.
1) The author of "Julie’s Race" wrote the story in order to
A) describe how dogs stay warm in cold weather
B) tell about a dogsled race.
C) explain how cold it can be in winter.
D) entertain the reader with funny stories about dogs.
2) Where does the dogsled race take place?
A) in Antarctic
B) on a trac
C) in Alaska
D) in a field
3) What happened BEFORE the dogs began running?
A) The dogs pulled the sled slowly.
B) Julie and the dogs lined up at the starting gate.
C) The runner on Julie’s sled broke.
D) The dogs pulled the sled over hills and into valleys.
4) Read this sentence from the story.
Julie’s team of dogs was lined up at the starting gate.
What does team mean?
A) friends and family
B) to join together
C) many dogs
D) a group working together
5) Why did Julie and her dogs win the race?
6) Why don’t the dogs freeze in the cold weather?
A) Julie puts special booties on their feet.
B) They sleep by the fire at night.
C) Their thick fur coats keep them warm.
D) It doesn’t get very cold in Alaska.
7) Read this sentence from the story.
The dogs’ thick fur coats helped keep them warm in the cold wind and weather.
What does thick mean?
A) thin
B) hard
C) fat
D) skinny
8) What kind of person is Julie?
EX 66. Read the passage below and do the task that follows.
Our planet is a trouble! Almost everyday we seem to hear problems _____(1) the environment – and what a
list of problems! Pollution, acid rain, global _____(2), the destruction of rainforests and other wild habitats,
the decline and extinction of thousands of species of animals and plants, and so on.
Nowadays, most of us know that these threats exist and that humans have caused them. Many of us are very
worried _____(3) the future of our planet and unless we can find a way of _____(4) the problems we have
made then the environment will suffer even more.
It all sounds depressing – but we certainly mustn’t despair! Every one of us, whatever age we are, can do
something to help slow down and reverse some of the damage. We can not leave the problem-solving entirely
to the experts – we all have _____(5) for our environment. We must learn to live in a sustainable way, learn to
use our _____(6) resources which include air, fresh water, forests, wildlife, farmland and seas without
_____(7) them. As populations expand and lifestyle change, we must keep the world in good _____(8) so that
future generations will have the same natural resources that we have.
EX 67. Read the letter below and do the task that follows.
London,
March 15th, 2007
Dear Peter,
It was nice to get your letter. I intended to write to you _____(1) a long time, but I'm not very good at
_____(2) in touch.
I am glad you are enjoying your new job in Paris - it sounds interesting. _____(3) I last wrote to you I have
done a lot of different jobs. After I finished my course, I moved to different places to find a job but I wasn't
quite happy with any jobs I found. I once worked on a farm in Florida, _____(4) grapes. It was a _____(5) job
but I didn't like it very much. At the end of that summer I left for Spain and then for New York but I wasn't
happy _____(6) any place, so I moved back to London. Now I am working for an import-export company and
the work is quite _____ (7).
What are you doing for Christmas? How about coming to stay with us? I _____(8) that you stay at my house
because now we've got a big and comfortable house. I am looking _____(14) to hearing from you soon about
your _____(15). My dad sends his regards to you and your family. I hope to see you at Christmas.
Best wishes,
Lena
EX 68. Read and answer:
Carnegie Hall, the famous concert hall in New York City, has again undergone a restoration. While this is
not the first, it is certainly the most extensive in the building's history. As a result of this new restoration,
Carnegie all once again has the quality of sound that it had when it was first built. Carnegie Hall owes its
existence to Andrew Carnegie, the wealthy owner of a steel company in the late 1800s. The hall was finished in
1891 and quickly gained a reputation as an excellent performing arts hall where accomplished musicians gained
fame. Despite its reputation, however, the concert hall suffered from several detrimental renovations over the
years. During the Great Depression, when fewer people could afford to attend performances, the directors sold
part of the building to commercial businesses. As a result, a coffee shop was opened in one corner of the
building, for which the builders replaced the brick and terra cotta walls with windowpanes. A renovation in
1946 seriously damaged the acoustical quality of the hall when the makers of the film Carnegie Hall cut a
gaping hole in the dome of the ceiling to allow for lights and air vents. The hole was later covered with short
curtains and a fake ceiling, but the hall never sounded the same afterwards.
In 1960, the violinist Isaac Stern became involved in restoring the hall after a group of real estate developers
unveiled plans to demolish Carnegie Hall and build a high-rise office building on the site. This threat spurred
Stern to rally public support for Carnegie Hall and encourage the City of New York to buy the property. The
movement was successful, and the concert hall is now owned by the city. In the current restoration, builders
tested each new material for its sound qualities, and they replaced the hole in the ceiling with a dome. The
builders also restored the outer walls to their original appearance and closed the coffee shop. Carnegie has never
sounded better, and its prospects for the future have never looked more promising.
1) This passage is mainly about
A) changes to Carnegie Hall
B) the appearance of Carnegie Hall
C) Carnegie Hall's history during the Great Depression
D) damage to the ceiling in Carnegie Hall
2) The word "extensive" could be best replaced by which of the following?
A) fabulous
B) thorough
C) devoted
D) continuous
3) What is the meaning of the word "detrimental"?
A) dangerous
B) significant
C) extreme
D) harmful
4) What major change happened to the hall in 1946?
A) The acoustic dome was damaged.
B) Space in the building was sold to commer-cial businesses.
C) The walls were damaged in an earthquake.
D) The stage was renovated.
5) Who was Andrew Carnegie?
A) a violinist
B) an architect
C) a steel mill owner
D) mayor of New York City
6) Which of the following words could best replace the word "gaping"?

(A) small
(B) round
(C) vital
(D) wide
7) The word "fake" is most similar to which of the following?
(A) low
(B) false
(C) thin
(D) handsome
8) What was Isaac Stern's relationship to Carnegie Hall?
(A) He made the movie "Carnegie Hall" in 1946.
(B) He performed on opening night in 1891.
(C) He tried to save the hall, beginning in 1960.
(D) He opened a coffeeshop in Carnegie Hall during the Depression.
9) What was probably the most important aspect of the recent renovation?
(A) restoring the outer wall
(B) expanding the lobby
(C) restoring the plaster trim
(D) repairing the ceiling
Which of the following is closest in meaning to the word "unveiled"?
(A) announced
(B) restricted
(C) overshadowed
(D) located
10) (A) predicted the result
(B) probed the plans
(C) was told in advance
(D) was stimulated to act
11) How does the author seem to feel about the future of Carnegie Hall?
(A) ambiguous
(B) guarded
(C) optimistic
(D) negative
12) Which of the following would most likely be the topic of the next paragraph?
(A) a scientific explanation of acoustics and the nature of sound
(B) a description of people's reactions to the newly renovated hall
(C) a discussion of the coffee shop that once was located in the building
(D) further discussion about the activities of Isaac Stern in 1960
EX 69. Read the test and fill in each gap with one given word in the box.

working doing sound scientific adults


writers universities angry computers have
Today, computer companies sell many different programs for computers. First, there are programs for
(1)................ math problems. Second, there are programs for (2)................ studies. Third, some programs like
fancy typewriters. They are often used by (3)...................and business people. Other programs are made for
courses in schools and (4) ....................... And finally, there are programs for fun. They include word games
and puzzles for children and (5) ......................
There are many wonderful new computer programs, but there are other reasons to like
(6)............... Some people like the way computer hum and sing when when they are (7) ............ .
It is a happy sound, like the sound of toys and childhood. Computers also (8)............ ... lights and pretty
pictures. And computers even seem to have personalities. That may (9) ................ strange, but computers seem
to have feelings. Sometimes they seem happy, sometimes they seem (10) ............... It is easy to think they are
like people.
EX 70. Complete the passage below by filling in each blank with the correct form of the word in the
brackets:
The families in our streets are slowly being driven mad by the (1)(refuse) ...................................... of the
inhabitants of No. 13 to have any form of communication with them. The trouble started over what is known
as noise (2)(pollute) ............................................... Every evening, the (3)(think) ............................................
neighbors used to turn up the volume on the TV so loud that no one in the (4)
(neighbor) ..................................................was able to hear anything else. Not knowing what (5)
(act) .................................................... to take, local residents held a meeting to see if anyone had any (6)
(suggest) ..................................................... as to how to deal with the problem. A decision was (7)
(make) ..................................................... to send a number of people to talk to the family in No. 13 and ask
them (8)(polite) ................................................. to turn their music down after six in the evening.
Unfortunately, the visit did not turn out to be (9)(success) ..................................................., as the inhabitants
of No. 13 refused to talk to them. So on the (10)(advise) ....................................................... of local police, the
matter is now in the hands of the court.
EX 71. Read the passage and answer with true ( T ) / false ( F ) / not given ( N ):
There are about 3000 living languages in the world, but only six of them are the most important ones. Two-
thirds of the world’s population speaks those languages. More than 400 million people speak English as their
mother tongue. Another 400 million speak it as a second language. No one knows how many people speak it as a
foreign language. Chinese is the language with more speakers than English, but it is only the language for more
than one billion Chinese people. English is the official language on one-fifth of the land area in the world. It is
spoken in North America, Great Britain, Australia, and New Zealand. In South Africa and India it is one of the
official languages. In many countries, the textbooks in universities are written in English. More than three-fourths
of the world’s mail is composed in English. More than three-fifths of the radio stations broadcast programs in
English. More than half of the scientific and research journals are in English. English is the language of
international communication.
1. Chinese is the most important language. ....................
2. About 66.6 per cent of the world’s population speak six major languages. ....................
3. Vietnamese speak English as their first language. ....................
4. 75 per cent of the world’s mail is in English. ....................
5. 40 per cent of the world’s radio stations use English. ....................
6. English has more speakers than Chinese. ....................
7. People in India speak English as their second language. ....................
8. People in New Dehli cannot speak English. ....................
9. People in France speak English as their foreign language. ....................
10. People in Australia also speak English. ....................
EX 72. Fill in each gap with ONE suitable word.
Water is our life source . It makes up 70 percent (1) ……..............…our bodies, and the average person actually
spends 18 months of his life in the bath (2)……….........……….shower.
But we are only now learning (3) ……….............to look after water. Acid rain has polluted as many as
18,000 lakes and our seas and our rivers are (4) ……...........………..with waste products. It is now very expensive
to try to repair the damage (5)………......………….has been done .We have some hope for the future, though,
because new (6)……… …………..of water have been discovered. People living the Sahara Desert have (7)
………………fish swimming in deep undergrowth steams. Scientists also believe (8)………… ……is a huge lake
beneath London. If we have learnt anything from our mistakes, we will try to keep these new areas of water clean .
EX 73. Put the correct preposition in the blackets if necessary.
1. This job is a lot different ……..…. what I’m used to.
2. ……..…. hot afternoons, we used to go swimming in the river.
3. We can see a lot of stars in the sky ……..…. night.
4. They’ll visit the shrine of a Vietnamese hero ……..…. tomorrow.
5. Phuong Linh did all the work ……..…. her own.
6. He went to school ……..…. the age ……..…. seven.
7. When my father was young he had to go to school ……..…. foot.
8. We’ll wait ……..…. the rain stops.
9. Our school was named ……..…. a young hero, Phan Dinh Giot.
EX 74. Give the correct form of the words in the parentheses.
1.Everyone wants to live a long and (HEALTH) ……..…. life.
2. I’d love to come to your party but (LUCKY) ……..…. , I have to go to somewhere else.
3. We’re very impressived by the (FRIENDLY) ……..…. of yourtown’s people.
4. Some designers have (MODERN) ……..…. the Ao dai by printing lines of poetry on it.
5. Your trousers are so long that it needs (SHORT) ……..…. .
6. Their teacher used to punish them by (MAKE) ……..…. them stay behind after school.
7. We started our trip on a beautiful (SUN) ……..…. morning.
8. Thanks for your (ENJOY) ……..…. evening.
9. She has one of the biggest (COLLECT) ……..…. in Britain.
10. If you do not see the doctor, your condition will only get (BAD) ……..…. .
EX 75. Read the following passage carefully and choose the best answer
Every year students in many countries learn English. Some of these students are young children. Others are
teenagers. Many are adults. Some learn at school, others study by themselves. A few learn English just by hearing
the language in films, on television, in the office or among their friends. But not many are lucky enough to do that.
Most people must work hard to learn another language.
Many boys and girls learn English at school because it is one of their subjects. They study their own
language, and mathematics… and English. In England, America or Australia, many boys and girls study their own
language, which is English and mathematics… and another language, perhaps French or German or Spainish.
Many adults learn English, because it is useful for their work. Teenagers often learn English for their higher
studies, because some of their books are in English at the college or university. Other people learn English because
they want to read newspapers or magazines in English.
1. According to the writer ………………
A. only adults learn English. B. no children like to learn English.
C. English is useful only for teenagers. D. English is popular in most of the world.
2. Most people learn English by ………………
A. watching videos only. B. hearing the language in the office.
C. talking with foreigners. D. working hard on their lessons.
3. Many boys and girls learn English because ……………
A. English can give them a job. B. it is included in their study courses.
C. they are forced to learn it. D. they have to study their own language.
4. In America or Australia many school children study …………
A. English as a foreign language. B. English and mathematics only.
C. foreign languages such as : French, German and Spainish.
D. their own language and no foreign language.
5. Many adults learn English because ………..…………
A. it helps them in their work . B. they want to go abroad.
C. most of their books are in English. D. English is spoken in their office .
EX 76. Fill each of the numbered blanks in the following passage. Use only one word in each space.
_ One day in June I wanted to visit some friends in Quang Binh, which is a long way ………1……. my
home. It ………2……. about three hours to get there by train. I looked at the timetable and saw that. ………3….
was a train that left at 5 a.m in the morning. It arrived ………4……. Quang Binh at 8 a.m and so I could spend
the whole day with my friends. The night before my journey I slept badly ………5……. I was afraid I would not
wake up in time to catch the train. I woke up at least an hour before my ………6……. clock went off at 4
o’clock.
When I arrived at the railway station, I found that the train times had changed on the 1st of June. There was
no train at 5 a.m . I had to wait until 7 a.m. It was raining heavily and I had to wait on a very wet platform ………
7……. the train arrived.
At quarter to 10, we were only five miles from Quang Binh when the train stopped. After a ………7…….
minutes, the ticket collector told the passengers that there was flooding on the lines and we could not go any
further. He said a bus would come and take us to Quang Binh station. We waited and waited. The train had no
restaurant and I was very ………9……. and thirsty. When I finally arrived in Quang Binh it was not 10 a.m but 4
p.m and I had only two hours ………10……. my friends before it was time to go home.
EX 77. READ AND ANSWER:   
Every day on the radio, on TV, and in the newspapers, we hear, see, or read about many problems in the world.
Because of this we must think about these problems. We must also try to find a solution for them. Our lives
depend on this. For example, there are pollution problems.
Air pollution is the first kind. It mostly comes from fumes released from cars, airplanes, and trains. Also, factories
give out waste anywhere, even in the city where many people are living. The factory owners must know that
people don’t want to live in pollution that is dangerous for their heath, Nobody in this world wants to breathe
dirty air.
1 …………. helps us know many problems in the world
A. Transportation B. Media C. Air D Factories
2. We can infer from the passage that…………
A.the factories owners must live in the city B.the factories owners don’t want to the in pollution
C.factories should be in the city D.factories shouldn’t be in the city
3. Pollution is the cause of many………………
A. diseases B. trains C. fumes D.cars
4. The word “THEM” refers to………
A. newspapers B. factories C. problems D. people
5. The word “DIRTY” in the last sentence of the passage can be best replaced by………
A. clean B. pure C. harmless D. polluted
EX 78. Read and then answer
   From far out in space, Earth looks like a blue ball. Since water covers three-fourths of the
Earth’s surface, blue is the color we see most. The continents look brown, like small islands floating
in the huge, blue sea. White clouds wrap around the Earth like a light blanket. The Earth is shaped
like a sphere, or a ball. It is 25,000 miles around! It would take more than a year to walk around the
whole planet. A spaceship can fly around the widest part of the sphere in only 90 minutes.
Even though spaceships have traveled to the Moon, people cannot visit the Moon without special
suits. The Moon has no air or water. Plants and animals can’t live there either. Astronauts first landed
on the Moon in 1969. After that, there were six more trips to the Moon. They brought back Moon
rocks, which scientists are still studying. There are holes, or craters, all over the Moon’s surface.
Scientists believe that meteorites smashed into the Moon millions of years ago and formed the craters.
_T he Sun is the closest star to Earth. A star is a hot ball of burning gas. The Sun looks very big
because it is so close. But the Sun is just a medium-sized star. Billions of far-away stars are much
bigger than our Sun. The burning gases from the Sun are so hot that they warm the Earth from 93
million miles away! Even though the Sun is always glowing, the night here on Earth is dark. That’s
because the Earth rotates, or turns around, every 24 hours. During the day, the Earth faces the Sun.
Then we see light. During the night, the Earth turns away from the Sun. Then it faces the darkness of
space.
Each day we learn more about the Earth, the Moon, and the Sun.
1. Why is blue the color we see most when looking at Earth from outer space?
A) Because most of the Earth is covered in land.
B) Because the Sun’s rays make the Earth look blue.
C) Because most of the Earth is covered in water.
D) Because clouds wrap around the Earth.
2. Read this sentence from the story.
Scientists believe that meteorites smashed into the Moon millions of years ago and formed the craters.
What does formed mean?
A) hit
B) made
C) broke
D) stopped
3. Write one fact and one opinion from the article that tells what the Earth looks like from outer space.
………………………………………..………………………………………..
………………………………………..………………………………………..
………………………………………..…………………………………
4. What causes daylight on Earth?
A) The full Moon causes daylight.
B) Daylight is caused by the Earth facing away from the Sun.
C) The heat of the Sun’s rays causes daylight.
D) Daylight is caused by the Earth facing toward the Sun.
5. Which of the following sentences BEST describes the Sun?
A) The Sun looks small because it is so far from Earth.
B) The Sun is a ball of burning gases that gives the Earth heat and light.
C) The Sun is a small star.
D) The Sun is not as hot as it looks.
6. Why did the astronauts bring rocks back from the Moon?
A) Because they didn’t know if they would return to the Moon ever again.
B) Because they wanted to prove that they went to the Moon.
C) Because they wanted to remember how the Moon looked.
D) Because they wanted to study them and learn more about the Moon.
Describe some of the things we know about the moon.
7. What is the main idea of the article?
A) Plants and animals can’t live on the Moon.
B) Without the Sun we would have no heat or light.
C) We know a lot about the Earth, Moon, and Sun, but there is still more to learn.
D) From outer space, the Earth looks tiny, even though it is thousands of miles around.

EX 79. Read and then answer


HOW THE CHIPMUNK GOT ITS STRIPES

     Long ago, the Earth was covered in darkness. None of the creatures living there knew what daylight looked
like.
One day, all of the animals of the forest gathered together in a clearing. They wondered if it would be better to
remain in darkness, or if it would be better to also have light. Deer, Chipmunk, Raccoon, Wolf, Bear, and many
other creatures climbed to the top of the highest mountain. The mountain stood so tall that there were no trees on
its top, and it was covered only with rocks. Millions of stars blinked in the dark sky overhead. The biggest and
most powerful animal in the forest was the bear, and he was the first to reach the mountaintop. Bear stood on the
highest peak, looked out over the forest below, and argued for remaining in darkness. He said that the creatures of
the forest would be able to sleep better in darkness because there would be no light to keep them awake. Most of
the other animals were afraid, and they agreed with Bear. Raccoon said that he did not mind the darkness because
he was so smart that he could find plenty of food, even in the dark. Wolf was easy to please, too. She didn't mind
the darkness because she could howl in darkness or in light. But one animal did stand up to Bear. Chipmunk, the
smallest of the animals, argued that it would be better to have both light and dark. Chipmunk was very clever. As
Bear continued to argue for darkness, she made many good arguments for light.
Slowly, the night passed. Bear grew tired of talking, but Chipmunk chattered on and on, as if she had all of the
energy in the world. As the other animals dropped off to sleep, one by one, Chipmunk kept arguing. Finally, the
first sunrise ever seen by the animals appeared over the top of the mountain. They woke up and were amazed by
what they saw.
Chipmunk began to dance from rock to rock. Bear became angry because he didn't get his way. He roared loudly
and ran after Chipmunk. He chased Chipmunk all the way down the mountain. Bear was fast, and he reached out
his giant paw to grab Chipmunk. Chipmunk got away, but not before Bear managed to scratch her back with his
long claws. And that is why, to this day, you can see stripes on Chipmunk's back!
1. What is the theme of this story?
A) Stand up for what you believe in.
B) It is always better to go along with the crowd.
C) There is only one right way to do things.
D) Making decisions is easy.
2. What is Bear like?
A) He is a good listener.
B) He is used to getting his way.
C) He is unsure about what he wants.
D) He is well liked by the other animals.
3. Read this sentence from the story
Bear grew tired of talking, but Chipmunk chattered on and on, as if she had all the energy in the world.
What does chattered mean?
A) moved slowly
B) laughed
C) spoke quickly
D) argued
4. According to the story, why do chipmunks have stripes on their backs?
5. Where does the story take place?
A) in a barn B) on an island
C) near a lak D) on a mountaintop
6. What happened after Chipmunk's argument with Bear?
A) The animals saw their first sunrise ever.
B) Bear was happy that he got his way.
C) The animals continued to live in darkness.
D) Chipmunk fell asleep.
7. Read this sentence from the story.
Chipmunk, the smallest of animals, argued that it would be better to have both light and darkness.
What is an antonym for the word argued?
A) Whispered B) agreed
C) Jumped D) yelled
8. In what ways is Chipmunk different from the other animals? How does this help Chipmunk to win the
argument with Bear? What lessons can we learn from Chipmunk?
EX 80. Fill in each blank of the paragraph with ONE suitable word :
People have always dreamed of living forever, and although we all know this will (1)………..happen, we still
want to live as long as possible. Naturally, there are advantages and disadvantages to a long (2)……….
Firstly people (3)………..live longer can spend more time with their family and friends. Secondly, people who
have busy working lives look forward to a long, relaxing (4)……………, when they can do the things they have
never had time for. On the other hand, there are some disadvantages. Firstly, many people become ill and
consequently have to (5)…………time in hospital or become(6)……….on their children and friends . Many of (7)
……….find this dependence annoying or embarrassing. In additon to this, the (8)………people get, the fewer
friends they seem to have because old friends die or become ill and it’s often (9)……to make new friends.
To sum up, it seems that living to a very old age is worthwhile for people who stay healthy (10)…….to remain
independent and enjoy life.
EX 81. Decide which answer which best fits the space
A few days ago Paul phoned me and (1).......whether I(2).......looking after his dog when he(3)....... away. I
(4)......... that I didn't really like dog, but he said that he (5)......all his other friends, and that I (6)......his only
hope. He invited me round to his house (7)......... to meet the dog, and he told me that he (8) ..... dinner for me.
An hour later he phoned again and said that he (9) .......after all, so I (10)..... meeting the following day for
lunch. The next morning he cancelled this oppointment and, after he (11) ...... said that he (12).......the dog to
my house at 6.00. " I don't know exactly (13).........., " he said. " Could you tell me how (14).....there?" I
quickly said I (15)..... out and put the phone down. Luckily I haven't heard from him since.
1. A told B said me C. asked me D. spoke
2. A would mind B want C. like D. will help
3. A has gone B. went C. would go D. will help
4. A told him B. said him C. asked him D. replied him
5. A asks . would ask C. will ask D. had already
6. A am B. will Be C. had Been D. asked
7. A his evening B. that evening C. the evening D. in evening
8. A is cooking B. will cook C. would cook D. had cooked
9. A had to go out B. went out C. goes out D. has to go out
10. A had suggested B. would suggest C. suggested D. suggest
11. A has done B. told me sorry C. asked D. had appologized
12. A would bring B. had brought C. brought D. brings
13. A where is it B. if it is C. where it is D. how was it
14. A do I get B. I get C. I will get D. I am getting
15. A went B. go C. will go D. was going
EX 82. READ AND ANSWER:
According to top executives in the industry, cigarette smoking is merely a nice habit, to be compared with
chewing gum or drinking your morning cup of coffee, and is no more addictive than eating candies. But what
is in fact the difference between eating donuts and smoking cigarettes? It is one of possible obesity or possible
death. In the U.S. about 400,000 deaths a year can be attributed to cigarette smoking. Cigarette makers insist
that there is no proof that heart disease, even lung cancer, or any other disease, is actually caused by cigarettes.
They deny adding nicotine to cigarettes; they even deny nicotine is addictive. They say that if it was, how
could 40 million Americans have given up smoking in the last 20 years. They compare it to coffee drinking
and ask if coffee manufacturers areaccused of adding caffeine to their coffee. Whatever the facts are, there is
no doubt however that cigarette manufacturers do try to entice young people, even in their teens, to smoke, by
advertisements and promotions that create an image even more addictive than the nicotine in cigarettes.
1) What is the best title for the passage?
(A) The Habits of Americans
(B) Comparisons between Eating and Smoking
(C) Death from Smoking
(D) How Addictive Cigarette Smoking Is
2) What is the attitude of cigarette manufacturers to nicotine?
(A) It has to be added to cigarettes.
(B) It is not addictive.
(C) It is better than caffeine.
(D) It is not as good as advertising.
3) Why do cigarette makers compare cigarette smoking with coffee drinking?
(A) Because both are customary practices people do.
(B) Because they are both better than eating donuts.
(C) Because neither are really nice habits.
(D) Because both add an addictive substance to their product.
4) The word "entice" means?
(A) to offer free
(B) to attract
(C) to addict
(D) to show
5) The author implies in the passage that cigarette manufacturers do?
(A) try to avoid making cigarettes addictive.
(B) succeed in making cigarette smoking like eating donuts.
(C) have an addictive product.
(D) worry about how addictive their products are.
EX 83. Read the passage and answer the questions:
Halloween is a holiday celebrated on October 31. The night of 31 st October was believed in the past that dead
people appeared from their graves, and which is now celebrated in the US,Canada and Britain.It is a time when
children have parties, and ghosts. Many children dress up as movie characters or a favorite superhero. They make
lanterns out of pumpkins (large and round vegetables) from which the inside has been removed, and play “trick or
treat”, a traditional activity at Halloween, in which children dress in costumes and visit houses.At each house they
say “trick or treat”.This means that they will play a ‘trick’, or joke ,on the people in the house unless they are
given a ‘treat’,e.g. sweets or money.Most people prefer to give treats rather than having tricks played on them.The
practice of ‘trick or treat’ began in the US in the 1930s but is now comon in Britain also. But Halloween is not just
for children. Many adults enjoy showing off their costumes at Halloween parties!
Questions:
1. When is Halloween celebrated?
2. Do they still believe dead people appear from their graves on the night of 31st October?
3. What do children pretend to be when they dress up?
4. Tell the activities children do at Halloween.
5. Do they have the practice of ‘trick or treat’ in Britain?
6. How do children play ‘trick or treat’?
7. In order to avoid tricks, what do people do?
8. Where is Halloween celebrated now?
EX 84. Read the text:
 You can ride a moped when you’re 16 years old, but you can’t ride a motorbike or drive a car until you’re
17. You can’t drive a lorry or bus until you’re 21.
 You can’t leave school and get a job until you’re 16. you can do a part-time job when you’re 13, but you
can’t work for more than 25 hours a week, and you can’t work before 7 o’clock in the morning or after 7
o’clock in the evening.
 When you’re 18 you can leave home and you can get married. (You can do these things when you’re 16, if
your parents agree.) You can also vote in elections when you’re18.
 You can go into a pub when you’re 14, but you can’t buy an alcoholic drink until you’re 18. You can smoke
at any age, but you can’t buy cigarettes until you’re 16. You can’t gamble until you’re 18.
EX 85. You are what you eat
My name’s Betty and this is my husband, Phil. We both work in offices in London. We have breakfast at half
past seven. We don’t have a big breakfast, because we have to work. We usually have toast, coffee, and orange
juice.
For lunch we usually have a salad or soup and a sandwich. That’s about 1.30.
We have dinner at about half past seven. It’s a big meal of the day and we have meat or fish with vegetables and
potatoes, pasta, or rice. We have fruit or ice cream for dessert. We have a glass of wine with the meal. On
Saturday evenings we usually go to a restaurant for dinner at about eight o’clock. We both like Chinese and Italian
food.
I’ m Roy and this is my wife, Joan. We live on a farm, so we have to get up early, at about 5.30. We start the
day with a big breakfast- bacon, sausages, and eggs, with tomatoes and mushroom. We have toast, too, and two or
three cups of tea.
Answer these questions:
a. Who do work in offices in London?
b. What do Betty and her husband have for breakfast?
c. Where do Betty and her husband usually go to on Saturday?
d. Where do Roy and his wife live?
e. What time do Joan and his wife get up?
EX 86. Read the text:
Monika is a chalet girl. She works in the ski resort of Verbier in Switzerland. She looks after groups of skiers.
Her day always starts early. She gets up at 5 o’clock in the morning. At 5.30 she walks to the shop and buys some
bread for the guests’ breakfast. At 7 o’clock she makes some coffee and takes it to the guests in bed.
After breakfast the guests go skiing. Then Monika washes up, makes the beds, and tidies the chalet. She has a rest
at about 11 o’clock. She doesn’t make lunch for the guests, so in the afternoon she normally goes skiing for about
three hours. At 4.30 the guests come back and have some tea, cakes, and a glass of wine. Then Monika cooks the
evening meal.
The guests often go to a café or a club after dinner. Monika sometimes goes with them or meet her friends, but she
doesn’t usually go out, because she has to get up early in the next day. So she washes up and then she usually
watches TV for an hour. She goes to bed at about 10.00.
Monika never goes skiing on Saturday, because it’s always a very busy day. The guests leave in the morning and
then Monika has to clean the chalet. After that she goes to the supermarket and buys food and other things for the
week, before the new guests arrive.
Monika likes working as a chalet girl, because she meets a lot of different people. “But”, she says, “It’s a job, not
a holiday”.
True or False?
a. Monika goes to the shop at 5 a.m.
b. She goes to the shop by car.
c. The guests have coffee in bed.
d. Monika tidies the chalet in the morning.
e. She makes lunch for the guests at 12.00
f. The guests have cakes, tea, and wine at about five o’clock.
g. Monika usually goes to a club in the evening.
h. She goes skiing every day.
i. The new guests arrive on Saturday morning.
j. Monika likes her job.

EX 87. Reading and writing: Home sweet home


 We live in Yorkshire, on a farm in the country. The farm house is quite an old building. It’s about 250
years old, I think. There aren’t many houses near us. There’s s pub in the village about three miles away,
but we’re over twenty miles from the nearest town. That can be difficult for shopping, but we love it here.
It’s very quiet, there’s no traffic, and the view is wonderful!
 We don’t live in a house or a flat. We live on a houseboat. The boat looks small, but it’s quite big inside.
Apart from the bathroom, it’s only got one room, so we have to do everything there- eat, cook, sleep, and
watch TV. We usually keep our houseboat at Little Venice in London. We’re almost in the centre of the
city there. That’s great for clubs, pubs, and the theatre and so on. But the best thing about the houseboat is
that you can move and take your home with you.
 We live in a semi-detached house in a suburb of Manchester. We’re got a living room, a dinning room, and
a kitchen downstairs. There’s a loo downstairs, too. Upstairs there are three bathrooms and a bathroom.
Outside there’s a garage, a front garden, and a back garden. The house isn’t very big but we like it. It’s
convenient for shops and schools and things like that, and the neighbors are very friendly.
 I’m a student at Bristol University. I share a house with three other students. It’s quite a big house. We’ve
all got our own room. There’s a bed, a desk, a chair, and a wardrobe in the room. It’s got a washbasin and
the mirror in the corner, too. We all share the kitchen, the living room, and the bathroom. We sometimes
have arguments about things, like when someone spends too long in the bathroom, but we have a lot of
fun, too.
EX 88. Number the paragraphs in the correct order:
o Then suddenly the chairlift stopped and Luke slipped from the chair. Fran grabbed his hand and stopped
his just in time. She tried to pull the child back into the chair, but he was too heavy. “ Help! Help!” she
shouted.
o But Luke didn’t fall. The chairlift operator climbed up to the chair and help Fran to hold her son. About
five minutes later the police helicopter arrived and lifted Luke into the chair.
o People looked up and screamed. Luke was 20 meters above the ground. Someone telephoned for a
helicopter and people hurried to put coasts on the ground under the boy.
o It was a normal day at the Dreamland theme park in Wellington, New Zealand, and young Luke Mansen
wanted to g on the Skyrider chairlift. With his mother, Fran, he waited for a chair. When it arrived, he
jumped in and Fran followed him.
o Everybody watched and waited while up in the chair Fran talked to Luke: “Don’t look down, Luke. Hold
on”. Every second was like a lifetime and the children started to cry. ‘I can’t hold on’, he whispered.
‘Goodbye, Mum’.
o The chair moved out into the sunshine. It was a warm, sunny day, so there were thousands of people at the
park. As the chair lifted Fran and Luke up into the sky they look down, and Luke waved to people on the
ground. It’s was Luke’s first time in a chairlift and he was very excited.
EX 89. Read the text again and number these events in the correct order:
Torquay? But I said Turkey
KUMIKO TSUCHIDA is a Japanese professor and she works in Turkey at Istanbul University. Last week she
took a short holiday in London. She had a good times, and after a few days she packed her suitcase, checked
out of the hotel, and set off to catch her flight back to Istanbul.
At Paddington station she couldn’t find the train to the airport, so she asked a railway guard. Mrs Tsuchida
doesn’t speak very much English, and when she said “Turkey”, the guard thought that she said “Torquay”, a
seaside town in south- west England. So he directed her to the platform for the 8.15 train to Torquay.
Mrs Tsuchida got on the train and found a seat. The journey seemed very long, but when she asked the other
passengers “Turkey?” they all said that yes, she was on the right train for Torquay.
She arrived in the seaside town just after midnight, but when she got off the train, of course she didn’t
recognize anything. She didn’t know where she was. She didn’t have any English money and she was very
frightened.
Two hours later, the police found her. They provided a bed for her for the night and the next morning they
phoned the Japanese embassy. The embassy sorted out the problem. Then Mrs Tsuchida got the train back to
London, took a taxi to Heathrow airport, and caught another flight to Istanbul.
Before she left, she spoke to our reporter through an interpreter. ‘ I said “Turkey, Turkey”, but people didn’t
understand my pronunciation and they thought I said “Torquay”. But I enjoyed my visit and English people are
very helpful and kind.’
 She traveled to Heathrow airport.
 She caught a flight to Istanbul.
 She got on the train.
 She went to Paddington station.
 The police found her.
 A railway guard directed her to the Torquay train.
 She spoke to a reporter.
 The police phoned the Japanese embassy.
 She spend a few days in London.
 She left Turkey.
 She arrived in Torquay.
EX 90. Read the text and put these items in the correct column:
Tamara Adam is fifteen years old. She normally catches the bus to school at 8.30. She meets her best friend,
Stacey Thomas, at the bus stop. Like all the other girls at Westin High School, Tamara wears the school uniform –
a green skirt and a white blouse, with green socks, a green tie, and a blue jacket. When she comes home in the
evening, she does her homework and watches TV.
But this week Tamara isn’t going to school. She isn’t living at home. She’s staying at an expensive London hotel.
And she definitely isn’t wearing her school uniform. She’s wearing the latest fashions.
Six months ago Tamara won a competition in Flair magazine. The prize was to be a model in a top fashion show
this week. So now it’s 11 o’clock on Wednesday morning. at this time Tamara usually walk across the school
playground to her next lesson. Nobody looks at her. She’s just another girl with her friend. But this week things
are very different. She isn’t walking across the playground. She’s walking along the catwalk at the London
Clothes Show. Hundreds of people are watching her and cameras are flashing.
Tamara’s mother, Barbara, is here, too. At the moment she’s sitting with Zoe Carson from the Ella Marie
modeling agency. Zoe: ‘ I think Tamara has got a great future as a model. Today models are like film stars. They
have to work hard, they can earn million.’
What does Tamara think? ‘ Well, I’m enjoying this week. I’m meeting a lots of new people and there are parties
every night. I’ve got my GCSE exams in June and I’m working hard for them, but after that, well, I think I’d
really like to be a model.’
Barbara is worried: ‘ Tamara’s only 15 years old, and yesterday Ella Marie offered her a contract for £50,000 a
year. That’s a lot of money. But things can change so quickly. This year you’re number one and next year nobody
wants you. But nobody can take your education away from you.’

appear at a fashion show go to school


do her homework meet new people
walk across the playground walk along the catwalk
meet her friends wear the latest fashions
nobody/ look at her go to the parties
hundreds of people/ watch her live at home
stay at a hotel wear a uniform
Tamara’s normal life This week

EX 91. Read the text : Aliens


“Are we alone in the universe?” Or is there life on other planet? Do the creatures from space visit the
Erath? Many people believe that they do. Anna Clarke is one of those people. This is her story.
It happened on 21 November 1990. Anna Clarke suddenly woke up. It was 3.15 a.m. She got up, walked to
the window, and looked out. There was a strange object like a huge wheel above the garden. At the centre of the
wheel there was a bright white light. The light started to move closer. Anna wanted to run away, but she couldn’t
move. Then she heard a voice inside her head. It said: “Don’t worry. Follow the light”. The light pulled her
downstairs and through the front door. When she was outside, the light pulled her up into the wheel.
Three small brown creatures appeared. They were naked. Their arms and legs were thinner than a person’s. On
each hand there were only three fingers and no thumb. Their heads were large with a small nose and mouth, and
they had huge, black eyes. The creatures took Anna to a larger room and put her on a kind of table.
Another creature came into the room. It was taller and fairer than the others. The creature pushed something
into Anna’s neck below her right ear. Anna looked into the creature’s eyes and she didn’t feel any pain. Something
seemed to pull her thoughts from her head. She felt very tired and she fell asleep.
When she woke up, she was in her bedroom again and it was morning. She remembered the light and the
creatures. She thought it was just a dream. But then she noticed that her feet were dirty, and when she looked in
the mirror she saw a small cut on her neck just below her right ear.
Do you believe Anna’s story? Perhaps you do, perhaps you don’t. However, many people believe that similar
things happened to them. David E. Jacobs, an American write, studied sixty of these stories. The most amazing
thing about the stories was that the sixty people didn’t know each other, but they all described the same creatures.
How do you feel about Anna’s story now?
Answer these questions:
a. Why did Anna wake up?
b. How did the creatures get Anna into the spaceship?
c. What was strange about the creatures’ hands?
d. What was different about the fourth creatures?
e. What did the creatures do to Anna?
f. Did she feel any pain?
g. How did Anna know that it wasn’t a dream?
h. What is interesting about David E. Jacob’s work?
EX 92. Read the text: The Year of the Tiger.
It was the last day of the US Masters golf tournament in Augusta, Georgia. As the slim, young man walked
up to the eighteenth hole he took off his baseball cap and waved to the crowd. A few minutes later the ball fell
into the hole and the crowd went wild. Tiger Woods was the new champion.
At 21 Tiger was the youngest ever champion, and he won with the best ever score. He was also the first black
golfer to win a major tournament. Everybody wanted to know him. He appeared on several TV shows. The
President of the United States, Bill Clinton, called him up and the sports company, Nike, offered him a contract
for $40 million.
Tiger (his real name is Eldrick) Woods was born on 30 December 1975 in Cypress, California. His mother,
Kultilda, was from Thailand. His father, Earl, was American army officer and a keen golfer. As a baby Tiger
watched his father, and he started to play golf as soon as he was old enough. When he was three years old he was
already a very good player, and he soon started to win tournaments. In 1991 he became the youngest ever Junior
champion. In November 1996 he became a professional, and less six months later he beat the best players in the
world to win the US Masters.
Some people think that Tiger is going to be even better in the future. ‘He’s going to win more tournaments, of
course,’ says golf commentator, Jeff Binns. ‘But Tiger Woods is more important than that. He hits the ball harder
and further than anyone else. He young and good-looking, too, so millions of kids are going to take up golf now,
and they’re all going to want to play like the Tiger’.
Answer the questions: Who or what are these?
Earl Woods
Cypress
Kultilda Woods
Jeff Binns
Eldrick Woods
Bill Clinton
Augusta, Georgia
Nike

EX 93. Read the text


Have you ever wanted to do something different? And have you ever wanted to learn more about our planet?
Five years ago Will Slade read about an organization called Earthwatch. Earthwatch finds volunteers for
expeditions to study and explore different parts of the world.
Will decided to join an expedition to study elephants in Africa. ‘I wasn’t sure about it before I went,’ say
Will. ‘But in fact I really enjoyed every minute of the expedition. We slept in tents and we cooked our own food,
but it was great to see the elephants and all the other animals there. Sometimes we didn’t find any, but most days
we are lucky.’
Will has now been or three more expeditions. He has counted birds in the rain forests of South America. He
has planted trees in Europe and he has studied whales in the Pacific Ocean.
‘On my last expedition I was in Hawaii. We were on a small island. We got up at 6 a.m, every morning and
had our breakfast. Then we went out in the boats and we looked for whales. We spent most of the day in a boat.
We photographed the whales, counted them, and we recorded their songs. Whales sing to each other, you know.
In the evening we put all the data into a computer. It was hard work, but we had a lot of fun, and I learnt a lot,
too.’
Earthwatch has found volunteers for hundreds of expeditions. They’re all ordinary people- male and female,
young and old, teachers, students, office workers, engineers. Volunteers have explored the forests and rivers of
south-east Asia. They’re dug up dinosaur bones in North America and they’re studied ancient civilizations in
Central America and Australia.
Here’s Will again. ‘I’ve enjoyed all the expeditions, and I’ve seen some fantastic places. How many people
have slept on a beach, climbed a mountain, or seen a whale? This world is such a beautiful place, but it’s
disappearing fast. It wasn’t comfortable
We have to learn more about it if we’re going to save it.’
EX 94. Complete the passage using the words below:

already during the limit feel


time every all can
Understanding their how
Pocket calculators are as cheap to buy as a pair of shoes, and as essential to thousand of school children as a
pencil and rubber. School has tried to (1) ……… their use by allowing them in science lesson to save (2)
……… but by not allowing them in mathematics classes. But when children do (3) ……… homework, a
pocket calculators is a need. But nowadays in Britain calculators (4) ……… be used freely in school
examination, and (5) ............ in many schools the only sound to be heard (6) ……… a mathematics
examination is the sound of children tapping out their answers.
Many parents (7) ……… that their children are not being given a basic (8] ……… of the facts and figures.
The process of addition, division, subtraction and multiplication is part of training in logic which is important
for (9) ……… child. It teaches them to take a series of steps necessary in coming to a conclusion. With a
calculator, the child can reach (10) ……… right answer, but have no idea at all (11)……… he got them.
EX 95. Fill in each blank with one suitable word
Like any other university, the open University can give you a degree. However, you don't have to (1)…………
working to study. It can also open up a whole variety (2) ……….. interest.
If you have (3) ……….. studied before, you will enjoy the special new pleasure of (4) ……….. your
knowledge. You will make friends of (5) ……….. kinds. You may also (6) ……….. that your qualification
provides new career opportunities.
You don't actually (7) ……….. to the Open University for lectures, but study at home, using television, radio
and computer software. You can (8] ……….. one class a month if you wish at an Open University center. Of
course there are exams to take as in (9) ……….. university. If you (10) ……….. like to know more, all you
have to do (11) ……….. complete the form below. It could be the start of (12)………..wonderful new period
in your life.
EX 96. From each number, pick out the word whose underlined part is pronounced differently from
the others.
1. A. equal B. fashion C. champagne D. match
2. A. only B. cotton C. cross D. economic
3. A. baggy B. minority C. style D. symbol
4. A. design B. ghost C. clothing D. strong
5. A. casual B. sale C. sleeveless D. slit
6. A. baggy B. fashion C. label D. casual
7. A. economy B. occasion C. freedom D. logical
8. A. encourage B. young C. proud D. enough
EX 97. From each number, pick out one word which has the stress on the first syllable.
1. A. baggy B. design C. equal D. poetry
2. A. inspiration B. fashionable C. modernize D. symbol
2. A. casual B. embroider C. designer D. occasion
4. A. convenient B. encourage C. material D. modernize
EX 98. Circle the word that the underlined part is pronounced differently from the others:
1. A. washed B. considered C. booked D. hoped
2. A. go B. game C. give D. large
3. A. given B. risen C. whiten D. ridden
4. A. blood B. tool C. noon D. spoon
5. A. watching B. matches C. kitchen D. machine
6. A. meal B. bread C. meat D. reason
7. A. beds B. doors C. students D. plays
8. A. cough B. although C. rough D. laugh
9. A. days B. ways C. says D. plays
10. A. banyan tree B. entrance C. paddy D. bamboo
EX 99. Choose the best answer:
1. I really enjoyed the disco. It was great, (isn’t it/ is it/ wasn’t it/ weren’t it)?
2. When I looked round the door, the baby (is sleeping/ was sleeping/ slept/ were sleeping) quietly.
3. If I (is/ are/ were/ would be) you, I’d take some rest before the exam tomorrow.
4. The meeting (isn’t/ won’t/ doesn’t/ hasn’t) be held this coming Sunday after all.
5. Don’t forget (locking/ for locking/ to lock/ lock) the door when you go out.
6. We couldn’t go out (because/ owing/ as of/ because of) the storm.
7. The couple (who/ whose/ which/ people) divorce was in the newspaper has got married again.
8. One of the students in my class (were/ was/ have been/ are) off sick today.
9. (In spite of/ Although/ However/ In spite) it was raining heavily, we went out without a raincoat.
10. I wish I (had listened/ would listen/ listened/ have listened) to him. Now it’s too late.
11. She spoke quietly to him (if/ because/ although/ so that) nobody else could hear a word.
12. The Ritchter system is named after the person (who/ whom/ which/ when) invented it.
13. She asked me where I (come/ came/ to come/ coming) from.
14. (On/ At/ In/ By) Easter, kids receive lots of chocolate Easter eggs in USA.
15. Let’s go for a walk, (will/ shall/ do/ must) we?
EX 100. Read the following passage and then decide which sentence is True or False.
Many people now think that teachers give students too much homework. They say that it is unneccessary for
children to work at home in their free time. Moreover, they argue that most teachers do not properly plan the
homework tasks they give to students. The results is that students have to repeat tasks which they have already
done at school.
Recently in Greece many parents complained about the difficult homework which teachers gave to their
children. The parents said that most of the homework was a waste of time, and they wanted to stop it. Spain
and Turkey are two countries which stopped homework recently. In Denmark, Germany and several countries
in Europe, teachers cannot set homework at weekends. In Holland, teachers allow students to stay at school to
do their homework. The children are free to help one another. Similar arrangements also exist in some British
schools.
Most people agree that homework is unfair. A student who can do his homework in a quiet and
comfortable room is in a much better position than a student who does his homework in a small, noisy room
with the television on. Some parents help their children with their homework. Other parents take no interest at
all in their children’s homework.
It is important, however, that teachers talk to parents about homework. A teacher should suggest
suitable tasks for parents to do with their children. Parents are often better at teaching their own children!
*True or False?
1. Arccording to the writer, many parents would like their children to have less homework.
2. Parents think that students shouldn’t have to do a lot of homework in their leisure time at home.
3. A lot of homework has not been planned properly, according to many parents.
4. Students say that most teachers give homework which is diffierent from the work they have done in class.
5. Greek parents thought their children’s homework was too easy.
6. Children do not have any homework now in Turkey.
7. In some countries in Europe teachers are allowed to give children homework only at weekends.
8. Children can do their homework at school and help one another in some schools in Britain.
9. Only a small number of people think that homework is fair.
10. Teachers should advise parents about how to work together with their children at home.
EX 101. Read the following passage then answer the questions below
All holidays are wonderful for school children and of course for me, too. I like spending my holidays in the
country side. It is lucky my grandparents are living in a small village, not very far from my city. So, I usually go
there by bicycle. Taking a leisurely bicycle trip to the countryside is the best way of recreation.
I have a good time staying with my grandparents during my holidays. I always feel healthier when I come
back to my city for my school work.
1. What do school children think of their holidays?
2. Where does the writer like spending his holidays? Why?
3. What is the best way of recreation?
4. How does the writer feel after his holidays in the countryside?
5. Have you ever had such holiday before?
EX 102. Rewrite these sentences
1. Someone is using the computer at the moment. -> The computer ……………………………..
2. “Do you grow your own vegetables?” I asked. -> I asked a farmer …………………………..
3. Mary was wearing the red dress. It was made in France. -> Mary was wearing the red dress …………..
4. We went out to the beach in spite of the heavy rain. -> Although …………………………………..
5. Shall we go to the disco tonight? -> What about ………………………………..
EX 103. Complete the following sentences using the word given
1. Mai/ wish/ she/ can/ live/ wwork/ a big city.
 …………………………………………………………………………………………………………………….
2. Lan/ like/ chatting/ Internet,/ she?
 …………………………………………………………………………………………………………………….
3. daughter/ hate/ have/ get up/ early.
 …………………………………………………………………………………………………………………….
4. Hoa/ best/ student/ this class.
 …………………………………………………………………………………………………………………….
5. Lan/ usually/ practise/ listen/ English programs/ TV.
 …………………………………………………………………………………………………………………….

EX 104. Choose the correct word or phrase for each space from the list below :
anything, are capable, even, keep, owing to, perform, raise,
recover from, regardless of, relax, something, take up, weave
A hobby can be almost (1) …………………………… a person likes to do in hi s spare time.
Hobbyists raise pets, build model ships, (2)………………… baskets or carve soap figures. They watch birds, hunt
animals, climb mountains, (3)................... flowers, fish, ski, skate, and swim. Hobbyists also paint pictures, attend
concerts and plays, and (4) ……………………………. On musical instruments. They collect everything from
books to butterflies, and from shells to stamps.
People (5)................... hobbies because these activities offer enjoyment, friendships, knowledge, and
relaxation. Sometimes they (6)................. yield financial profit. Hobbies help people (7).................. after period of
hard work. Hobbies also offer interesting activities for person who have retired. Anyone, rich or poor, old or
young, sick or well, can follow a satisfying hobby, (8).................... his age, position, or income.
Hobbies can help a person's mental and physical health. Doctors have found that hobbies are very valuable
in helping patients (9) ……………………………… physical or mental illness. Hobbies give bedridden or wheel-
chair patients something to do and provide interest that (10) ………………... them from thinking about
themselves .
EX 105. Read the text below and think of the word that best fits each space. Use only one word in each
space :
THANKSGIVING
Thanksgiving is a truly American holiday. Nobody should have dinner alone on that day. That's what
Thanksgiving is all (1) ……………………… The first immigrants, who were called Pilgrims, (2).................. to
America from Europe about three hundred and fifty years (3) …………………… They were good friends with
(4) ………………………Indians and they tried ( 5 ) ……………….........corn and other stuff together but it was
really hard farming at that time. When they got the first harvest, the (6) …………………….. and the Americans
had a big dinner together to (7)................ God for their good luck. And the food the American eat now (8)
…………………………… Thanksgiving are the same kinds of things that the Pilgrims ate: turkey, com, and
squash. The date is (9) ……....................every year but it is always the fourth Thursday of November. In Canada,
they (10) ………………………… it in October.
EX 106. Choose the word whose underlined part is pronounced differently from the rest.
1. A. cell B. cube C. city D. rice
2. A. brought B. ought C. thought D. though
3. A. hear B. spear C. dear D. swear
4. A. promise B. devise C. surprise D. realise
5. A. bush B. pull C. brush D. push
EX 107. Choose the word with the position of the stressed syllable different from the rest.
1. A. career B. avoid C. manage D. advise
2. A. burglary B. employment C. courageous D. dependent
3. A. computer B. expression C. instantly D. appliance
4. A. population B. electricity C. popularity D. concentrate
5. A. curriculum B. enthusiast C. arrangement D. temperature
EX 108. READ AND CHOOSE THE BEST ANSWER: Martin Luther King, Jr
Martin Luther King, Jr., is well known for his work in civil rights and for his many famous speeches, among
them is his moving "I Have A Dream" speech. But fewer people know much about King's childhood. M.L., as
he was called, was born in 1929 in Atlanta, Georgia, at the home of his maternal grandfather. M.L.'s
grandfather, the Reverend A.D. Williams, purchased their home on Auburn Avenue in 1909, twenty years
before M.L. was born. The Reverend Williams, an eloquent speaker,
played an important role in the community since so many people's lives centered around the church. He
allowed his church and his home to be used as a meeting place for a number of organizations dedicated to the
education and social advancement of blacks. M.L. grew up in this atmosphere, with his home being used as a
community gathering place, and was no doubt influenced by it.
M.L.'s childhood was not especially eventful. His father was a minister and his mother was a musician. He
was the second of three children, and he attended all- black schools in a black neighborhood. The
neighborhood was not poor, however. Auburn Avenue was the main artery through
a prosperous neighborhood that had come to symbolize achievement for Atlanta's black people. It was an area
of banks, insurance companies, builders, jewelers, tailors, doctors, lawyers, and other black-owner black-
operated businesses and services. Even in the face of Atlanta's segregation, the district thrived. Dr. King never
forgot the community spirit he had known as a child, nor did he forget the racial prejudice that was a
seemingly insurmountable barrier that kept black Atlantans from mingling with whites.
1. What is this passage mainly about?
(A) the prejudice that existed in Atlanta (B) Martin Luther King's childhood
(C) M.L.'s grandfather (D) the neighborhood King grew up in
2. The word "eloquent" in line 7 means most nearly
(A) powerful (B) active
(C) romantic (D) fascinating
3. The word "gathering" in line 12 could best be replaced by
(A) picking (C) exciting
(B) learning (D) meeting
4. As used, the word "eventful" in line 15 is closest in meaning to which of the following?
(A) valued (C) admirable
(B) memorable (D) emotional
5. In line 20, the word "it" refers to which of the following?
(A) achievement (C) segregation
(B) neighborhood (D) services
6. According to the author, blacks in King's neighborhood were involved in
all the following businesses and services EXCEPT
(A) dentistry (C) law
(B) medicine (D) banking
7. The word "tailors" in line 21 describes people who are associated with which of the following trades?
(A) flower arranging (C) garment making
(B) shoe making (D) book binding
8. According to the author, King was influenced by
(A) community spirit (C) his mother
(B) black lawyers (D) his speeches
9. The word "thrived" in line 23 refers to which of the following?
(A) achieved (B) surrendered (C) flourished (D) held
10. As used in line 25, which of the following is closest in meaning to the word "seemingly"?
(A) apparently (C) inexplicably
(B) inevitably (D) hastily
11. The word "mingling" in line 26 could best be replaced by which of the following?
(A) interfering (C) consuming
(B) gargling (D) associating
12. According to the author, M.L.
(A) had a difficult childhood (B) was a good musician as a child
(C) loved to listen to his grandfather speak (D) grew up in a relatively rich area of Atlanta
EX 109. READ AND ANSWER:
How many things can you see in the night sky? A lot! On a clear night you might see the Moon,
some planets, and thousands of sparkling stars.  You can see even more with a telescope. You might
see stars where before you only saw dark space. You might see that many stars look larger than
others. You might see that some stars that look white are really red or blue. With bigger and bigger
telescopes you can see more and more objects in the sky. And you can see those objects in more and
more detail. But scientists believe there are some things in the sky that we will never see. We won't
see them with the biggest telescope in the world, on the clearest night of the year.
That's because they're invisible. They're the mysterious dead stars called black holes.
You might find it hard to imagine that stars die. After all, our Sun is a star. Year after year we see it
up in the sky, burning brightly, giving us heat and light. The Sun certainly doesn't seem to be getting
old or weak. But stars do burn out and die after billions of years.  As a star's gases burn, they give off
light and heat. But when the gas runs out, the star stops burning and begins to die.As the star cools,
the outer layers of the star pull in toward the center. The star squashes into a smaller and smaller ball.
If the star was very small, the star ends up as a cold, dark ball called a black dwarf. If the star was
very big, it keeps squashing inward until it's packed together tighter than anything in the
universe. Imagine if the Earth were crushed until it was the size of a tiny marble. That's how tightly
this dead star, a black hole, is packed. What pulls the star in toward its center with such power? It's
the same force that pulls you down when you jump — the force called gravity. A black hole is so
tightly packed that its gravity sucks in everything — even light. The light from a black hole can never
come back to your eyes. That's why you see nothing but blackness.  So the next time you stare up at
the night sky, remember: there's more in the sky than meets the eye! Scattered in the silent darkness
are black holes — the great mystery of space.
1) According to the article, what causes a star to die?
A) As its gases run out, it cools down.
B) It collides with other stars.
C) It can only live for about a million years.
D) As it gets hotter and hotter, it explodes.
2) Read this sentence from the story.
They're the mysterious dead stars called black holes.
What is a synonym for the word mysterious?
A) ordinary C) strange
B) bright D) common
3) In what ways is our Sun the same as other stars? How is it different from a dead star?
Which of the following statements is NOT a fact?
A) Black holes are dead stars. C) Black holes are invisible.
B) Black holes have gravity. D) There is nothing as mysterious as a black hole.
4) What happens AFTER a star dies?
A) It becomes invisible. C) It burns up all of its gases.
B) It falls to Earth. D) It becomes brighter and easier to see.
5) What might happen to our Sun billions of years from now? 
Why can't you see light when you look at a black hole?
A) because most black holes are so far away
B) because the gravity of a black hole is so strong that it sucks  the light inward
C) because as the star's gases burn, it stops giving off heat and light
D) because as a star cools, its outer layers pull in toward its center
6) Read this sentence from the story.
The Sun certainly doesn't seem to be getting old or weak.
What is an antonym for the word weak?
A) thin C) strong
B) tired D) big
EX 110. WORD FORM:
1. I really don't think he has the ________ to do this job. (ABLE)
2. The professor explained his ideas with great ________. (CLEAR)
3. Do you know what the ________ of the river is? (DEEP)
4. He is a very ________ carpenter. (SKILL)
5. He ________ to hit me if I didn't hand over my money. (THREAT)
6. He drives so ________ that he is sure to have an accident. (CARE)
7.I am ________ he will succeed in his chosen career. (CONFIDENCE)
8. Craig's old jacket was so ________ that he had to buy a new one. (WEAR)
9. The ring was not valuable; in fact it was almost ________. (WORTH)
10. During his speech, he kept on ________ his tie. (STRAIGHT)
EX 111. WORD FORM:
1. The water in this area is ________ and should not be drunk. (PURE)
2. A successful business needs good ________. (ORGANISE)
3. Alison's ________ made it hard for her to speak in public. (SHY)
4. He used all his ________ to force the door open. (STRONG)
5. He was too ________ to tell his teacher about the stupid mistake. (SHAME)
6. The police are interested in the sudden ________ of the valuable painting. (APPEAR)
7. I believe you because I know you are ________. (TRUTH)
8. I feel so ________ that I'm going to bed. (SLEEP)
9. The cost of the ________ must be paid by the buyer. (CARRY)
10. Recently health foods have increased in ________. (POPULAR)
EX 112. READ AND ANSWER:
It is very difficult to succeed in the music business; nine out of ten bands that release a first record fail to
produce a second. Surviving in the music industry requires luck and patience, but most of all it requires an
intricate knowledge of how a record company functions. The process begins when a representative of a
company's Artists and Repertoire (A & R) department visits bars and night clubs, scouting for young, talented
bands. After the representative identifies a promising band, he or she will work to negotiate a contract with
that band. The signing of this recording contract is a slow process. A company will spend a long time
investigating the band itself as well as current trends in popular music. During this period, it is important that a
band reciprocate with an investigation of its own, learning as much as possible about the record company and
making personal connections within the different departments that will handle their recordings. Once a band
has signed the contract and has finished recording an album, the Publicity and Promotions department takes
over. This department decides whether or not to mass produce and market the band's album. Most bands fail to
make personal contacts in this second department, thus losing their voice in the important final process of
producing and marketing their album. This loss of voice often contributes to the band's failure as a recording
group.
1) Which of the following statements best expresses the main idea of the passage?
(A) Nine out of ten bands fail to produce a second record.
(B) It is important for a band to have an intricate knowledge of how a recording company works.
(C) Making personal connections will help the band in the final decisions about the promotion of their
album.
(D) The main factors in a band's success are luck and patience.
2) What is the meaning of the word "release"?
(A) distribute (B) pay for
(C) overturn (D) itemize
3) The phrase "an intricate" could be best replaced by which of the following?
(A) a fleeting (B) a straightforward
(C) an extraneous (D) a detailed
4) According to the passage, the initial contact between a band and a recording company is made by
(A) the band's manager (B) a band member
(C) an AR representative (D) the Publicity and Promotions department
5) The word "reciprocate" could be best replaced by which
of the following?
(A) commence (B) respond
(C) practice (D) confirm
6) The word "investigation" is closest in meaning to which of the following?
(A) production (B) betrothal
(C) credential (D) examination
7) Which of the following words is most similar to the word "handle"?
(A) touch (B) control
(C) manipulate (D) protect
8) What is the meaning of "takes over"?
(A) takes charge (B) takes pleasure
(C) takes advice (D) takes blame
9) The author mentions that a band's success is dependent on all of the following factors EXCEPT
(A) having patience
(B) making personal contacts with people in the company
(C) understanding how a record company functions
(D) playing music that sounds like music of famous bands
10) According to the passage, the Publicity and Promotions department
(A) has the final decision in producing an album
(B) handles the recording arrangements for the band
(C) sends representatives to look for new talent
(D) visits bars and night clubs
11) The author uses the phrase "losing their voice" to illustrate that they
(A) are forbidden to speak
(B) are unable to visit
(C) have no representation
(D) are too shy to express their desires
12) It can be inferred from the passage that
(A) the music industry is full of opportunities for young bands
(B) the AR department has a very large staff
(C) most bands do not fully understand how record companies operate
(D) the cost of recording an album is very expensive
EX 113. Fill in each numbered blank with ONE word chosen from the list below to complete the
meaning of the passage :

Baked single rich restaurants besides includes leaves they children favourite

BIRTHDAYS IN ENGLAND

Birthdays are celebrated with friends and family, and are usually geared towards children although adults
also enjoy celebrating their birthdays. A lot of people who have young…(1)... no longer have birthday parties
at home.Now they go to fast-food…(2)… like McDonald’s or Wacky Warehouse because children’s
playgrounds are there. Also …(3)…hold them in pubs or community centers which can hold more guests.
Food served at these parties usually…(4)…a birthday cake, which is sometimes called “ Fortune
Telling Cake” because small charms are mixed into the batter before the cake is.. .(5)…These little objects
all have symbolic meanings. For instance, a slice of cake with a coin in it means you will be…(6)….Trifle,
sausage rolls, cheese and pineapple on a stick, cocktail sausages and sandwiches are other…(7)…party foods.
Today’s popular birthday gifts might be computer games like Nintendo, Sony or Sega…(8)…the traditional
gifts of books or clothing or money.

EX 114. Fill a suitable word in each blank to complete the passage. (10p)
1. accidents 2. fire 3. hot 4. burns 5. play
6. controls 7. catches 8. dangerous 9. fingers 10. drops

Many thousands of children have……(1)……..in their homes.As a result some children die. The most
common accidents are with……(2)…..and hot water. Small children often touch pots of……(3)…… water
on the stove. The pots fall over and the hot water falls on the children and…..(4)…… them. Some children
like to ……(5)……..with fire. They enjoy striking matches or throwing things on fire to make it burn
brightly. If the fire gets too big, it gets out of…(6)……..Then the house……(7)…… fire. It is very …(8)
……..to play with matches. When a child strikes a match, the flame soon burn near his……(9)…....Then
he……(10)……..the match on the floor.Many houses catch fire in this way.

EX 115. READ AND DO THE TASK


Our oceans are becoming extremely polluted. Most of this pollution comes from the land, which means it
comes from people. Firstly, there is raw sewage, which is pumped directly into the sea. Many countries, both
developed and developing, are guilty of doing this. Secondly, ships drop about 6 million tons of garbage into the
sea each year. Thirdly, there are oil spills from ships. A ship has an accident and oil leaks from the vessel. This not
only pollutes the water, but it also kills marine life. Next, there are waste materials from factories. Without proper
regulations, factory owners let the waste run directly into the rivers, which then leads to the sea. And finally, oil is
washed from the land. This can be the result of carelessness or a deliberate dumping of waste./.

HOW THE OCEAN IS POLLUTED


Firstly: ......................................................................................
Secondly: ..................................................................................
Thirsly: .....................................................................................
Next: .........................................................................................
Finally: .....................................................................................
EX 116. Complete the text with a suitable word for each space:(2ms)
More than hundred years ago, the term “environmental pollution” (1)………………. quite strange to (2)
………………….. They lived healthily, drank pure water, and breathed (3)………………….. air. Nowadays,
the situation is quite different. People all over the world are worried about things that are happening to the
environment. Actually it is man that is destroying the surroundings with many kinds of (4)
…………………….We know that motobikes and cars emit dangerous gases that cause poisonous air and
cancer, but no one wants to travel (5)………………………foot or by bike. Manufacturers know that wastes
from factories make water and soil (6)…………………….., but they do not want to spend a lot of their
money on treating the (7)……………………….safely. Scattering garbage is bad for our health, but no one
wants to (8)…………………………time burying it. Is it worth talking a lot about pollution?
EX 117. Read the text and fill only one suitable word in each blank. (2pts)
Bill Gates is a very important person in the (1)………. industry. He (2)………. been chief
executive officer of Microsoft Corporation for several years. He is (3)……….. the richest person in the
world. How did he do it?
He learned a lot (4)……… his parents. While Bill was going to school, his father went to college,
got a degree, and (5)…….. a successful lawyer. From this, Bill learned that you have to work hard (6)………
you want something. His mother was a very busy teacher, but she enjoyed (7)……… to parties. From this, he
learned something else: (8)…….. you want to work hard and play hard, you have to (9)…….. . a schedule.
When Bill was young, he spent a lot of time alone. (10) …….. most of his friends were playing,
Bill read all of the World Book Encyclopedia and (11)……… it when he was eight years old.
Bill’s childhood was not all work, (12)…….. . He (13) ………. to play a lot of sports- swimming,
water, skiing, tennis. He was very serious (14) ……… sports. He loved winning and he hated (15)……… .
When Bill got older he spent more and more (16)…….. working- and playing- (17)…….. a computer.
Before he was twenty, Bill developed the world’s first computer (18)……… for the personal
computer. Once he was thinking about the future, he realised something important. He thought that every
home was (19) ………. to have a computer, and every computer would (20)………. software- his software.
He said, “I’m going to make my first million dollars on software by the time I’m 25.” And he did.
EX 118. Read the passage and fill in each space with ONE appropriate word from the words given in
box. Two words aren’t used

words, food, rid, work, comes, accounts, example,


up, starve, living, unless, know, let’s, hope, means

The need for money (0) comes from the fact that different people in society produce different things.
This (1)___________ that people depend on each other for goods and (2)___________. Let us take the
(3)___________ of a farmer who produces more food than he requires and a carpenter who earns a
(4)___________ by selling the tables and chairs that he has made. It will be obvious that (5)___________
some means of exchange is found, the farmer will not be able to get (6)___________ of his surplus food and
the carpenter, having nothing to eat, will (7)___________ to death! Clearly, the possible means of exchange
for them to use will be barter – in other (8)___________, to exchange a certain amount of one kind of goods
((9)___________ say flour) for a certain amount of another (tables or chairs, in this case).
Obviously, barter can (10)___________ only in a very simple society. In an advanced society one can’t go
around carrying things in the (11)___________ that he can exchange them for the right things he needs. So,
we need something that will stand for the goods and services that we want to exchange. That
(12)___________ for the origin of money.
EX 119. Read the passage below and select the best answer.
HUNGRY POP STARS
Valerie Jones runs a company called Peanuts whose job is to look after pop stars and pop groups when
they go on tour. She is the person who feeds the stars and she’s been doing it for the past ten years.
When the stars are playing at a festival, Valerie may have to cook for up to a thousand people which
includes all the crew and the people who work backstage. She erects a marquee-a huge tent-and the food is served
buffet style from a central serving area. She has to cater for different tastes, so there are normally four or more
choices of menu. She also has to look after people who may be on special diet or some singers don’t eat dairy
food before a concert.
She drives an enormous truck full of kitchen equipment and hires at least three walk-in refrigerators, a
dishwashing unit and portable cabins which act as storerooms and office.
All the bands have to queue up to be served and everyone has to have a meal ticket. The stars are usually
more relaxed when they are eating as no one is bothering them for autographs, although Valerie says that
sometimes the security men and the stars’ managers are more trouble than the stars themselves.
There are certain things which she always has to keep in stock like herbal teas and her own particular
mixture of honey, lemon and ginger which singers like to keep in flasks on stage with them when they are
singing. Years ago bands used to drink quite a lot of alcohol, but these days they’re much healthier. Most
bands drink fresh fruit juice and prefer to eat salads.
A lot of people in the bands are quite young and they’re not used to very expensive food, so Valerie
prepares plain food unless a band sends her a “rider”. This is a list of special requirements. When people are
tired, unwell or homesick they like to have familiar” comfort” food so she keeps a stock of people’s
requirements just in case. As a result of all this Valerie says she has become an expert shopper and in less
than an hour in a supermarket she can spend £1000.
A lot of bands won’t eat before a concert because they’re too nervous, so Valerie and her staff can
end up working very long hours as they have to be around to provide what people want at two or three in the
morning. One thing Valerie has noticed is that the more mad a band is on stage, the more normal they are
when they are off it. She says she is amazed at the change in behaviour. A really wild singer can turn out to
be really quiet and polite off stage.
1. Peanuts is the name of
A. a pop band B. a pop star C. a pop concert D. a company
2. Valerie has to provided a range of food because
A. people are very fussy about what they eat
B. people are used to eating in restaurants
C. there is such a wide variety of preferences
D. there is such a demand for special menus
3. The singers are less nervous when they are eating because
A. their security men are with them B. there are no fans hanging around
C. their managers fuss over them D. the bands enjoy eating together.
4. Why does Valerie have to keep a supply of certain drinks?
A. the bands really rely on a special recipe B. the bands like to drink alcohol
C. the bands prefer herbal tea to coffee D. the bands take fruit juice on stage
5. What do most bands like best to eat?
A. rich food B. cheap food C. junk food D. simple food
6. What does “just in case” in paragraph 5 refer to?
A. Valerie’s supply of more expensive food
B. Valerie’s list of “riders” from the different bands
C. Valerie’s supply of special food for various people
D. Valerie’s understanding of people feeling sick
7. Why do you think Valerie has become an “expert shopper”?
A. She has a lot of money to spend each week
B. She has learnt to find what individuals want
C. she has to buy as much as possible for £ 1000
D. She has to shop very quickly in a supermarket
8. Why is a band likely to be hungry after playing?
A. They feel more relaxed after the concert
B. They work long hours with little food
C. They only have a snack before a concert
D. They like to wait until they eat together
9. What does Valerie think about the singers?
A. They are completely crazy on and off stage.
B. They behave differently on and off stage
C. They are less rude when they are off stage
D. They are normally more noisy on stage
10. How does Valerie feel about the behaviour of the singers
A. surprised B. angry C. uncomfortable D. pleased
EX 120. Read the passage carefully and then answer the questions below it
People who know Salley Ride say she is intelligent and confident. She also thinks deeply about things, but
she is not always serious. She is fun and enjoys humor but she is in a hurry to get on to more important things.
At high school she became interested in science. At university she studied English and Physics.
After completing her studies at Stanford University, she worked as a researcher. In 1977, she was one of the 1,000
women and 7,000 men who applied for the astronaut training program when she was 25. She was one of the 35
women who were accepted to start the program in 1978. As part of her training, she learnt to be a jet pilot and a
flight engineer. She married during her training. Her husband, Havley, is also an astronaut. They do not plan to
have any children yet.
a. What did she do in 1977?
……………………………………………………………………………………………
b. Have they got any children?
………………………………………………………………………………………
c. Give the fact from the text to prove that Salley Ride is intelligent
……………………………………………………………………………………………………………….
EX 121. Use the given words in the box to complete the text below
which carry because to
is the richest in
Brunei is one of the smallest counties in the world. Its population ……… (1) ……… only 250,000
and most of them live ………… (2) …………… Bandar Seri Begawan (the capital city with ……………… (3)
………………… longest name in the world). It is also the ………… (4) ………… country in Asia and maybe the
richest country in the world ……… (5) ………… it has a lot of oil and gas ……………… (6) it exports to Japan.
Every week huge tankers …… (7) …… oil and gas from the oil-fields of Brunei …………… (8)
………………… Japan.
EX 122. Read the following passages carefully, then choose the ONE best answer to each question
At the age of sixty-five, Laura Ingalls Wilder began writing a series of novels for young people based on
her early experiences on the American frontier(linh vuc) . Born in the state of Wiscosin in 1867, she and her
family were rugged (khó nhọc) pioneers(người đi đầu) . Seeking(theo đuổi ) better farm land, they went by
covered wagon(xe ngựa) to Missouri in 1869, then on to Kansas the next year, returning to Wisconsin in 1871, and
traveling on to Minnesota and Lowa before settling(bố trí) permanently(cố định) in South Dakota in 1879.
Because of this continuing moving, Wilder's early education took place sporadically(không thương xuyên) in a
succession of one-room schools. From age thirteen to sixteen she attended school more regularly although she
never graduated.
At the age of eighteen, she married Almanzo James Wilder. They bought a small farm in the Ozarks,
where they remained for the rest of their lives. Their only daughter, Rose, who had become a nationally known
journalist, encouraged her mother to write. Serving as agent and editor, Rose negotiated(đàm phán thương lượng)
with Harper's to publish her mother's first book, Little House on the Big Woods. Seven more books followed, each
chronicling her early life on the plains. Written from the perspective(triển vọng, tiền đồ) of a child, they have
remained popular with young readers from many nations. Twenty years after her death in 1957, more than 20
million copies had been sold, and they had been translated into fourteen languages. In 1974, a weekly television
series, "Little House on the Prairie ", was produced based on the stories from the Wilder books.
1. What is the main topic of the passage?
A. American pioneer life B. Children's literature
C. A weekly television series D. Wilder' s career
2. Laura Ingalls Wilder began writing novels
A. when she was a child on the frontier B. right after she moved to the Ozarks
C. when she was a young mother D. after her sixty-fifth birthday
3. The author mentions all of the following events in the life of Laura Ingalls Wilder EXCEPT
A. She went west by covered wagon B. She graduated from a one-room school
C. She married Almanzo Wilder D. She had one daughter
4. Wilder's early education took place
A. with great success B. for a long time
C. at irregular intervals D. in a very efficient way
5. Written from the perspective of a child, they have remained popular... THEY refers to
A. the plains B. many nations C. more books D. young readers
6. It can be referred from the passage that
A. Laura Ingalls Wilder wrote scripts for the television series.
B. the Wilders were not happy living in the Ozarks.
C. Wilder's daughter was not a successful writer.
D. the Wilders books have a universal appeal.
7. Laura Ingalls Wilder died in
A. 1967 B. 1957 C. 1947 D. 1977
8. Laura Ingalls Wilder's family lived forever in South Dakota since
A. 1867 B. 1869 C. 1879 D. the age of 65
EX 123. Use the correct form of the words in brackets to complete sentences
1. That company has 2000 ............ (EMPLOY)
2. I don’t know why they call him a ............. (LIE)
3. The old hotel we used to stay at has been .................... (MODERN)
4. Conservationists believe that we should preserve the ......rainforests in the world. (TROPIC)
5. The little village is very quiet and .............at night. (PEACE)
6. Women nowadays have more ..........to participate in social activities (FREE)
7. The .........of the new guest caused trouble to my aunt. (ARRIVE)
8. She cut herself and it's ...........quite badly. (BLOOD)
9. His parents are very ............of him (PRIDE)
10. Every week, there are two ...........from Ha Noi to Nha Trang. (FLY)
11. Bell experimented with ways of transmiting ...........over a long distance. (SPEAK)
12. The tiger wanted to see the farmer's ............. (WISE)
13. The scouts do the ................work (VOLUNTEER)
14. Nowadays, scouting is popular ................ (WORLD)
15. The scouts also help the .............and street children, (OLD)
16. We're very impressed by the ..........of your town's people. (FRIEND)
17. I must clean this ..............floor. ( DIRT)
18. It was ............not to write down the address. ( FOOL)
19. Is it possible to .........between a hobby and an interest? (DISTINCT)
20. Thousands of people have been made..........by the war (HOME)
EX 124. Choose the best answer (a, b, c or d ) for each space
There has been a revolution in the world of newspapers. Not many years ..1....., newspapers were being
produced using techniques unchanged for ..2.. hundred years.
The journalists gave their stories to a typist, who prepared them for an editor, who passed them
on...3... the printer. The printer who was a ..4.... skilled man, set up the type. ......5... was then collected to
make the pages. When the pages were complete, the printing machines could be...6.......
Nowadays what ...7.....? The journalists type their stories into a coputer. The ...8 ..... checks their
spelling, plans the page, shapes the articles. When the pages are ready, another computer may control the
printing.
........9.. can be no doubt about it., producing a newspaper in an entirely different ..10.......now
1.a.before b. after c.ago d. yet
2.a.a b.some c.an d.over
3.a.to b.by c.through d.with
4.a.hardly b.mostly c. partly d.very
5.a.They b.Which c.This d.All
6.a.switched b.started c.stopped d.moved
7.a.gives b.occurs c.goes d.happens
8.a.computer b.editor c.typist d.printer
9.a.It b..There c.You d.We
10.a.skill b.work c.management d.business
EX 125. Choose the word whose part underlined is pronounced differently from the others in each
group.
A. needed B. worked C. stopped D. booked
A. thick B. thorough C. healthy D. weather
A. like B. lively C. live D. life
A. cook B. blood C. school D. look
A. feat B. great C. seat D. fear
A. flame B. hate C. page D. adventure
A. sugar B. settle C. surprise D. sentence
A. helped B. liked C. watched D. hated.
A. good B. book C. foot D. food
A. lie B. height C. weight D. pie

EX 126. Use the correct form of the words in brackets to complete sentences
He is ..........active in spite of his old age. ( WONDER)
.............is a very serious problem in many countries (EMPLOY)
He suffered from constant ................. ( SLEEP)
After the explosion, only two people were left ....... (LIVE)
Her husband's death made her life ........... (MEAN)
There are more and more people concerning about ..........pollution (ENVIRONMENT)
Is Buddhism the country's ..........religion of Thailand? (OFFICE)
I watch the news everyday because it's very ........... (INFORM)
I try to speak clearly when I meet a ............ ( FOREIGN)
Many pupils at ..........school have already choosen a career. ( SECOND)
EX 127. Use the correct form of the words in brackets to complete sentences
1. ..............., the step mother was very cruel to Little Pea. ( FORTUNATE)
2. everyone was .............at the soccer match. (EXCITE)
3. Many rural areas in Vietnam have been ........... (URBAN)
4. Tim and Shanon want to visit an ...........institute in Vietnam ( OCEAN)
5. Should English be a ...........foreign language in Viet Nam? (COMPEL)
6. Trung's sense of humor ..........him from other students. ( DISTINCT)
7. He is looking ..............thinner ( APPRECIATE)
8. The .........sat there asking for money. ( BEG)
9. She got a.........letter from her boss ( PERSON)
10. We had to take out a .........from the bank to buy the car. ( LEND)
EX 128. Read the text and choose the correct word for each space: English around the world
English is the first language of many people in countries outside the United Kingdom.
When you...1... speakers of English from around the world, you ...2.... notice that they do not all speak in the same
way. There are also some ......3...... in the words they use, including the names of ...4.... objects that are part of
everyone’s daily life. But although pronunciation and ...5... are not the same everywhere, it is interesting that
English speakers...6.... opposite sides of the world can understand ...7... other quite easily. It does not seem to ...8...
where they learnt the language. And of course this is one reason why speakers of other languages are keen ...9...
learning English too. If you know English, you are more...10... to be able to study or work in all sorts of exciting
places, such as the United States or Australia.
1. A- recognise B- meet C- find D- attend
2. A- originally B- strangely C- curiously D- immediately
3. A- mistakes B- corrections C- changes D- differences
4. A- common B- popular C- favourite D- general
5. A- reading B- composition C- dictation D- vocabulary
6. A- of B- in C- from D- at
7. A- each B- one C- the D- some
8. A- mine B- care C- matter D- worry
9. A- by B- on C- to D- for
10. A- likely B- probably C- possibly D- luckily
EX 129. CHOOSE THE WORD OR PHRASE THAT BEST FITS EACH SPACE
In child development there is an important phenomenon that shows very clearly the process of
preparation for the future: play. (1) ______ to popular belief, its importance should never be underestimated.
Games are not the haphazard creations of parents or educators. They should be seen as educational (2) ______
and as stimuli for the child’s (3) ______, imagination and life skills. Every game is a preparation for the future.
The manner in which children (4) ______ a game, their choice of game and the importance they (5)
______ upon it, show their attitude and relationship to their environment and how they relate to their
(6) ______ human beings. Whether they are hostile or whether they are friendly, and particularly whether they
show qualities as leaders, are clearly (7)______ in their play. In observing children at play we can see their
whole attitude towards life; play is of the (8) ______ importance to every child.
But play is more than preparation for life. Games are (9)______ communal exercises that enable children to
develop their social feeling. Children who avoid games and play are always (10) ______ to the suspicion that they
have not (11)______ satisfactorily to life. These children gladly withdraw from all games, or when they are sent to
the playground with other children usually (12) ______ the pleasure of others. Pride, lack of (13)______ and the
consequent fear of ‘getting it wrong’ are the main reasons for this behaviour. In general, by watching children at
play, we can determine (14)______ great certainty the (15) ______ and quality of their social feeling.
1.  A. In contrast B. Contrary C. According D. Due
2.  A. means B. sources C. tools D. aids
3.  A. psyche B. physiology  C. mindset D. nerves
4.  A. operate B. approach C. process D. experience
5.  A. give B. accord C. place D. lay
6.  A. fellow  B. contemporary C. present D. peer
7.  A. distinct B. evident C. noticeable D. marked
8.  A. great B. utmost C. prime D. most
9.  A. without doubts B. in all C. by far D. above all
10. A. attached B. open C. prone D. likely
11. A. survived B. adjusted C. changed D. grown
12. A. spoil B. damage C. vanish D. worsen
13. A. maturity B. egoism C. self-esteem D. development
14. A. on B. in C. with D. for
15. A. level B. scale C. scope D. extent
EX 130. Use the correct word form: (3ms)
1) My new car is more . . . . . . . . . . . . . . . . than the one I had before. (ECONOMY)
2) He didn't feel happy because he worked . . . . . . . . . . . . . . .(SUCCESS)
3) My colleagues are very pleasant, but the manager is a little. . . . . . . . . . . . (FRIEND)
4) We must make a . . . . . . . . . . . . . . . . .about where to go. (DECIDE)
5) Lucy got . . . . . . . . quickly and went for work. (DRESS)
6) We all looked . . . . . . . . . . . . . . after the summer holiday. (HEALTH)
7) The children are more . . . . . . . . . . . . . . . . in cartoons. (INTEREST)
8) . . . . . . . . . . . . . . . . ., the weather was so bad that we couldn’t go out. (FORTUNE)
9) I hope there won’t be too much . . . . . . . . . . . . . . in getting a work permit. (DIFFICULT)
10) That . . . . . . . . . . . . . . has composed a lot of beautiful songs. (MUSIC)
EX 131. Read the passage and choose the correct answer.
Thomas Edison was born in Milan, Ohio, in 1847. His family moved to Port Huron, Michigan, when he
was seven years old. Surprisingly, he attended school for only two months. His mother, a former teacher,
taught him a few things, but Thomas was mostly self-educated. His natural curiosity led him to start
experimenting at a young age.
Thomas Edison lit up the world with his invention of the electric light. Although the electric light
was the most useful, it was not his only invention. He also invented the record player, the motion picture
camera, and over 1,200 other things. About every two weeks he created something new. He worked 16 out of
every 24 hours.Sometimes he worked so intensely that his wife had to remind him to sleep and eat.
Thomas Edison died in 1931, in West Orange, New Jersey. He left numerous inventions that
improved the quality of life all over the world.
1. Thomas Edison was ______.
A. a discoverer B. a teacher C. an explorer D. an inventor
2. In 1854 Edison’s family ______.
A. moved to Port Huron, Michigan B. bought a new house in Milan, Ohio
C. decided to settle in Milan, Ohio D. sent him to a school in New Jersey
3. The word “self-educated” in the passage mostly means ______.
A. “having been well taught” B. “having had good schooling”
C. “having taught himself” D. “having had a high education”
4. Edison died at the age of ______.
A. 76 B. 84 C. 47 D. 74
5. Which of following statements is NOT true about Edison?
A. He invented the motion picture camera. B. He didn’t go to school at all.
C. He made numerous inventions. D. He worked very hard.
EX 132. Read the passage and choose the correct answer.
New research says 35 percent of the deaths of children worldwide are caused by hunger. The research is
from poor to middle-income countries in Africa, Asia and Latin America.
Robert Black from the Bloomberg School of Public Health at Johns Hopkins University in Maryland was
the leading writer of the research. He says more than 3.5 million mothers and children under five die in poor
countries each year because of hunger.
He says more than two million children die from underdevelopment, either before or after birth. Millions of
others who survive face a lifetime of disabilities or early death. And the effects are not just physical. Poor brain
development can limit economic success as children become adults. Then the cycle of poverty and hunger often
repeats for their children.
Doctor Black says hungry children are also more likely to have conditions like high blood pressure and
heart disease as adults. He says the studies show that food programs need to place the greatest importance on
the first two years of life. Hungry children can suffer the whole life damage from age two.
So it is high time to improve their diets. Diets should include foods rich in vitamin A and other useful
things. The researchers say early help like these could reduce child deaths by 25%.
1) What is the main idea of the third paragraph?
A. Poverty and hunger have influenced all children's lives in the world.
B. Hunger has some bad effects on children's physical and mental development.
C. Ever year over two million children die of hunger after birth.
D. Poverty and hunger can limit people's success.
2) What can we learn from this passage?
A. If a child is hungry at childhood, he will be disabled in the future.
B. Criticism is the best medicine for hunger.
C. The results from the research is not satisfactory.
D. Generally speaking, hungry children have less chance of catching heart disease when they grow up.
3) Which period is the most important for children's physical development?
A. Before birth.                             B. Between birth and the age of two.
C. From age two to five.                D. Under the age of five.
4) Which of the following is the most suitable title for this passage?
A. Hunger is children's biggest enemy.                      B. How to prevent hunger.
C. A latest survey about hunger all over the world.    D. Time to improve our living conditions.
EX 133. Read the passage and choose the correct answer.
Justin's bedroom was so full of flat bicycle tires, bent tennis rackets, deflated basketballs, and games with
missing pieces that you could barely get in the door. His parents pleaded with him to clean out his room.
"What use is a fish tank with a hole in the bottom?" his father asked. But Justin simply smiled and
repeated his motto, "Never throw anything out, you never know when it might come in handy."
When Justin was away from home, he always carried his blue backpack--a smaller version of his
bedroom--a place to store the many objects that he collected. It was so worn and stretched that it hardly
resembled a backpack anymore.
Justin had earned a reputation for figuring things out and getting people out of otherwise hopeless
situations. Many of his classmates and neighbors sought him out when they needed help with a problem. On
the first day of school, his friend Kenny, came looking for Justin.
"Do you think you have something in your bag that could help me remember my locker combination?" he
asked. "I lost the piece of paper it was written on. I have a science class in two minutes and if I'm late on the
first day it'll make me look bad for the rest of the year." Kenny looked really worried.
"Relax," Justin said, taking his backpack off and unzipping the top. "Remember how you borrowed my
notebook in homeroom to write the combination down? Well, I know how we can recover what you wrote."
He took the notebook and a soft lead pencil out of his bag. The page that Kenny had written on had left
faint marks on another page in the notebook. Justin held the pencil on its side and rubbed it lightly over the
marks. Slowly but surely the numbers of the locker combination appeared in white, set off by the gray pencil
rubbings.
"That's amazing!' Kenny said. "I owe you one." And he dashed off to open his locker.
It was just another day in the life of the boy whose motto was "Never throw anything out, you never know
when it might come in handy."
1) Why is Justin's room such a mess?
 A. He always forgets to clean it.           B. He never throws anything away.
C. He has no time to clean it.               D. He shares the room with his brother.
2) In what way is Justin's backpack a smaller version of his bedroom?
A. He uses it as a place to store objects.
B. He uses it to carry his books and sports equipment.
C. His parents tell him to clean it all the time.
D. He's had it for as long as he can remember.
3) The underlined phrase "pleaded with" means
A. Ignored          B. Asked                    C. Pushed              D. Ordered
4) How does Kenny feel toward Justin?
A. Annoyed.       B. Disinterested.          C. Grateful.            D. Angry.
EX 134. Read the passage, then complete the passage with the words in the box:

college difficult evening hard language


newspapers subjects teenagers useful studies
Every year many people in the world learn English. Some of them are young children. Others are (1) ……
Many are adults. Some learn at school, others learn in (2) …………………. classes. A few learn English by
themselves or just by hearing the (3) …………………. on television or among their friends. Most people must
work (4) …………………… to learn English. Why do these people want to learn English? It isn’t (5)
…………………………. to answer that question. Young children learn English at school because it is one of their
(6)…………………. Many adults learn English because It is (7) ……………………………… for their work.
Teenagers often learn English for their higher (8) …………………………, because some of their books are in
English at (9)………………………. or university. Other people learn English because they want to read (10)
……………………. or magazines in English.
EX 135. Fill in each gap in each sentence with the correct form of the word in capital letters.
1. My teacher usually ...................... me to take part in the IOE. COURAGE
2.Some young people today want to have a strange ........................ APPEAR
3.Thank you for a very ..................... evening at your house . ENJOY
4. Although both ............................ were good at English, we decided on the one with more experience. APPLY
5. Ants and bees are described as .........................insects. INDUSTRY
6. He doesn’t agree with the .............................there is life on other planets. BELIEVE
7. He promised to come but he is very ................................... RELY
8.I’d prefer not to work but I don’t have much .......................... CHOOSE
9.It’s always.......................... to hear other people’s point of view. INTEREST
10. He had real great ………………. to design this Ao dai INSPIRE
EX 136. Complete the sentences using the correct form of the words in brackets.
1. He is a famous……………………. (ACT)
2. He ....................... for his wrongdoings. (APOLOGY)
3. He plays soccer …………..............……….……… (BEAUTY)
4. This is the biggest……………..in the city. (BUILD)
5. Bill Gates is a famous .......................... (BUSY)
6. Hoa has a fine …………..............……….……… of stamp. (COLLECT)
7. You must ................................. the answer you choose. (DARK)
8. She welcomes us…………..............……….……… (DEEP)
9. There is no …………..............……….……… between my answer and his. (DIFFER)
10. We have a lot of …………..............……….……… in learning English. (DIFFICULT)
11. Nam is very…………..............……….……… and heavy. (ECONOMICALLY)
12. I felt very …………..............……… when I was a member of our school (EXCITE)
team.
13. The is an…………..............……….……… football match this afternoon. (EXCITE)
14. Her smile is very ............................ (FASCINATE)
15. He speaks English ..................................... (FLUENCY)
16. We are very proud of our………..............………. (FRIEND)
17. He turns out to be the………..student in his class. (GOOD)
18. He leads his life ................... (HAPPY)
19. If you smoke, it is …………..............……….……… to you. (HARM)
20. He does exercise every morning, so he is very…………..............……….…… (HEALTH)
21. Smoking is bad for your............................... (HEALTHILY)
22. Her parents’ …………..............……….……… makes her very sad. (ILL)
23. News on TV is very ...................... (INFORM)
24. He gets bad grades because of his................... (LAZILY)
25. What is his ............................? (NATION)
26. Every man loves ................................ beauty. (NATURE)
27. She had a lot of ............................ for Tet. (PREPARE)
28. He run very..............……….……… (QUICK)
29. It is cloudy and .......................... today. (RAIN)
30. We are interested in .......................... books. (READ)
31. Ha Long Bay is one of the ........................... of the world. (WONDERFULLY)
32. This…………has written many stories for children. (WRITE)
33. Water is ....................................... (COLOR)
34. A large number of .............................. comes to Nha Trang every year. (VISIT)
35. You will be late for the ................................ ( MEET)
36. Would you tell me some ......................... about the train, please? (INFORM)
37. Ask the …………..............… if you want to borrow books here. (LIBRARY)
38. That …………..............… plays a lot of instruments. (MUSIC)
39. Lan’s brother is a …………..............… . He plays it very well. (PIANO)
40. If the engine goes wrong, tell the …………............. (ENGINE)
41. My brother is a …………..............… at Vietnam National University in Hanoi (STUDY)
42. Phone the …………..............… if the lights don’t work. (ELECTRIC)
43. He is a …………..............… He travels a lot. ( JOURNAL)
44. Nam, you will be an …………..............… (ART)
45. Where is the …………..............… post office from here? (NEAR)
EX 137. Read the text then choose the correct answer
Computing is now at the same stage as printing was when the first printing presses were used. Before
printing presses were invented, only rich people like kings and dukes could afford to buy books. Often these
people were unable to read and hadn't enough time to learn. In any case, the books were so big that it was
difficult for anyone to relax with a book as we do today. They wanted books because they were expensive
and there was something magical about them. Only a few people were able to write, and it took an
extremely long time to write a book. Monks and other people who could write said ordinary people could
not learn to read.
The position with computers is very similar today. A few years ago, computers were very large and
expensive. Business managers and rich people ordered them but they didn’t know how to use them. In many
countries, however, the situation has now completely changed. Lots of people not only own microcomputers
but also know how to use them.
1) What happened before printing presses were invented?
A) Books were so big that people didn’t want to buy them.
B) Only intelligent and rich people could read books.
C) Only kings and dukes had enough money to buy books.
D) All rich people bought books.
2) Why were books wanted before printing presses were invented?
A) to show that the people who owned the books were rich and intelligent.
B) to show that the people who owned the books could write.
C) Because books were dear and magical.
D) Because books showed that the person who owned them was magical people.
3) What is the common feature of a book and a computer?
A) They were only for intelligent people.
B) They were too expensive to buy.
C) They were large and expensive at first.
D) Only people who could use them buy them.
4) What situation has through changed nowadays?
A) A lot of people are eager to buy computers
B) A lot of computers are sold
C) Even children can use a computer
D) A person who buys a computer knows how to use it.

You might also like